Vous êtes sur la page 1sur 131

www.MRCPass.

com

Rheumatology


elzohryxp@yahoo.com https://www.facebook.com/elzohryxp

May2012

www w.MRCPass.c com

Rheum matology

Rheumatol logyQ001 1

A 35 5 year old man m presen nts with a 6 month his story of arth hralgia, mouth ulcerat tion and eye irritation. On O examina ation, he ha ad some ulc ceration in t the mouth, bilaterally swollen stsandredu ucedrangeofmoveme entsofboth hknees. wris His investigatio ons showed d: white cel ll count 11 x 10^9/L, C reactive protein p 100 0 mg/dl, eumatoidfactornegativ ve. Rhe atisthelike elydiagnosi is? Wha
A.Reiterssyndrome e B.Sjogre enssyndrom me C.Ankylosingspondylitis etssyndrom me D.Behce E.Sarcoidosis

Answer:d)Beh hcetssyndr rome. Beh hcets syndrome is a multisystem m disorder characteris sed by recu urrent oral genital ulce eration, eye e lesions (an nterior and d posterior uveitis or retinal r vascu ulitis), skin lesions, (ery ythemanod dosum,papu ulopustular rlesionsfoll liculitis)and dapositivepathergyte est. The e pathergy phenomenon is considered an outstanding feature of o Behcet disease. d Following a ne eedle prick or intradermal injection with sa aline or dilu ute histami ine, the pun ncture site becomes inflamed and deve elops a sm mall sterile e pustule due to hyperactivityof o theskinto oanyintrac cutaneousinsult.

OralUlceration ninBehcet'sdisease
Dr.Khalid K Yusuf fElzohrySo ohagTeachin ngHospital 2012 2

www.MRCPass.com

Rheumatology

RheumatologyQ002
A 33 year old female with systemic lupus erythematosis has arthralgia involving her upperlimbs.Shealsohasabutterflyfacialrashandarashonthetrunk. UrinedipstickshowsnoProteinuriaorhaematuria.Herrenalfunctionisnormal. Whichoneofthefollowingmedicationsismostappropriate?
A.Methotrexate B.Prednisolone C.Azathioprine D.Hydroxychloroquine E.Cyclosporin

Answer:D)Hydroxychloroquine. NSAIDsandhydroxychloroquineareusedforskininvolvementandarthritis. NSAIDs are used for mild disease. Hydroxychloroquine is useful for disease not controlled by NSAIDS. Steroids are used in moderate to severe disease. Immunosuppressive treatments such as azathioprine and cyclophosphamide are used typicallywhenthereisrenalorcerebaldisease.

Dr.KhalidYusufElzohrySohagTeachingHospital2012 3

www.MRCPass.com

Rheumatology

RheumatologyQ003
A 75 year old woman with longstanding Rheumatoid arthritis has great difficulty walkingandcommentsthatbothherlegsarestiffandjumpy. Whatisthemostlikelycauseofthepresentation?
A.Ankylosingspondylitis B.Syringomyelia C.Osteoporosis D.Atlantoaxialinstability E.Disccompression

Answer:d)Atlantoaxialinstability. CervicalcordcompressionduetoatlantoaxialinstabilityisthemostlikelycauseofUMN nerveweakness.

Dr.KhalidYusufElzohrySohagTeachingHospital2012 4

www.MRCPass.com

Rheumatology

RheumatologyQ004
A 50 year old woman has severe rheumatoid arthritis. She is admitted with worsening breathlessness. She is currently on ibuprofen, methotrexate, celecoxib, and paracetamol. On examination,shehasfeaturesofrheumatoidchangesinherhandsandlookspale.There isnopalpablelymphadenopathyandnoabdominalmasses. Investigations showed that her Hb level is 7.8 g/dl with a MCV of 90, WCC 2.1 x 10^9/l andplatelets55x10^9/l. Reticulocytecountis0.3%(0.5%to1.5%). Whatisthelikelycauseoftheanaemia?
A.NSAIDuseandGIbleed B.Treatmentwithcelecoxib C.Treatmentwithmethotrexate D.Anaemiaofchronicdisease E.Felty'ssyndrome

Answer:c)Treatmentwithmethotrexate. The patient has an aplastic anaemia which can be caused by methotrexate or azathioprine,DMARDs(penicillamineorgold). Abnormally low reticulocyte count can be attributed to chemotherapy, aplastic anemia, perniciousanemia,bonemarrowmalignanciesandlowerythropoietinlevels.

Dr.KhalidYusufElzohrySohagTeachingHospital2012 5

www w.MRCPass.c com

Rheum matology

Rheumatol logyQ005 5
A65 5yearoldwoman w pres sentswithdry d eyesandadrymouth.Herinv vestigations sshow: ANA A strongly positive (1 1:1600), ant tiRo/SSA an ntibodies strongly s positive, rheu umatoid fact torpositive, ,IgGmarkedlyelevatedat42g/l (nor rmal<15g/ /l),IgMandIgAlevelsa arenormal. Wha atisthelike elydiagnosi is?
A.Mono oclonalgamm mopathyofunknown u sig gnificance B.Prima arySjogren's sSyndrome C.SystemicLupusEr rythematosu us D.Reiter'ssyndrome e E.PolyarteritisNodo osa

Answer:b)Prim marySjogre en'sSyndrom me. The eclinicalfeaturesandare a consiste entwithprim marySjgre ensSyndrom me. Hyp pergammaglobulinaem miaispresen ntin80%of findividuals s.ANA,AntiRo/SSAant tibodies are present in approximat tely 90% of f individuals s and there can also be e a weakly positive rheu umatoidfac ctor.

DrymouthseeninSjogren n'ssyndrom me

Dr.Khalid K Yusuf fElzohrySo ohagTeachin ngHospital 2012 6

www w.MRCPass.c com

Rheum matology

Rheumatol logyQ006 6
A 55 5 year old female f has been on longterm ste eroids for chronic obst tructive pulmonary dise ease. She co omplainsof fpain inher r rightgroin n radiatingd down theanteromedia althigh. Shehasanantalgicgait. On examinatio on of the hip, there is s decreased d range of movement especially flexion, abductionandinternalrot tation. Wha atisthelike elydiagnosi is?
A.Osteo oarthritis B.Rheumatoidarthr ritis C.Metastatichiples sion D.Avasc cularnecrosi isofthefem moralhead E.Hairlin nefracture

Answer:d)avascularnecrosisofthef femoralhea ad. In a patient on long term m steroids presenting with groin n pains radi iating to th he thigh asso ociated with h an antalgic gait and decreased range of movement of f the hip, th he most likelydiagnosis sisavascula arnecrosisofthefemo oralhead. In th his conditio on, MRI is th he most sen nsitive and specific tec chnique and d is useful for f early diag gnosis befo ore collapse e of bone occurs. CT scan and xray are useful to rule r out advanced disea ase ifdurationis notclear.Bonescanning s ismoresensi itivethan xraybut isno onspecific.

MRIshowingavascularne ecrosisofth hefemoralhead h


Dr.Khalid K Yusuf fElzohrySo ohagTeachin ngHospital 2012 7

www.MRCPass.com

Rheumatology

RheumatologyQ007
A 60 year old woman on treatment for longstanding rheumatoid arthritis presents with breathlessness. She complains of a dry cough. The oxygen saturation was found be 85% onair. Thechestxrayshowsadiffusebilateralinterstitialinfiltrate. Bloodculturesandsputumculturesarenegative. Whichdrugislikelytohavecausedthisadverseeffect?
A.Cyclosporin B.Cyclophosphamide C.Gold D.Methotrexate E.Sulphasalazine

Answer:d)methotrexate. Pneumonitis is a serious and unpredictable sideeffect of treatment with methotrexate (MTX) that may become lifethreatening. Chest radiography reveals a diffuse interstitial ormixedinterstitialandalveolarinfiltrate,withapredilectionforthelowerlungfields. Pulmonary function tests show a restrictive pattern with diminished diffusion capacity. Lung biopsy reveals cellular interstitial infiltrates, granulomas or a diffuse alveolar damagepatternaccompaniedbyperivascularinflammation.

Dr.KhalidYusufElzohrySohagTeachingHospital2012 8

www w.MRCPass.c com

Rheum matology

Rheumatol logyQ008 8
A 65 year old man prese ents has rec cently deve eloped an acutely a pain nful right kn nee. On examination,he h hadatem mperatureo of37.2Cahot h andswo ollenrightknee. k His white cell count c which h was raised d at 14 x10 0^9/l. A kne ee X ray sho owed reduced joint spac ceandchon ndrocalcino osis. Cult tureofaspiratedfluidshowed s nogrowth. Wha atisthelike elydiagnosi is?
A.Psoria aticarthropa athy B.Rheumatoidarthr ritis C.Pseud dogout D.Septicarthritis E.Osteo omalacia

Answer:c)pseudogout. The eclinicalpictureistypic calofpseud dogout.Calcificationo ofthearticu ularcartilage ewould be consistent and the cu ulture results excludes septic ar rthritis. The e diagnosis can be confirmed by y the pres sence of calcium pyrophosph p hate crysta als in join nt fluid dem monstrating gapositiveb birefringenc ceunderpo olarisedligh ht.

Calc ciumpyroph hosphatecr rystals


Dr.Khalid K Yusuf fElzohrySo ohagTeachin ngHospital 2012 9

www.MRCPass.com

Rheumatology

RheumatologyQ009
A 60 year old man presents with a painful swollen left knee as wellas pain and stiffness ofbothhishands. On examination his skin is pigmented. There is tenderness and swelling of the 2nd and 3rd metacarpophalangeal joints of both hands, he has hepatomegaly of 8 cm below the costalmargin. Hiskneeisswollenandaspirationofthejointyieldsturbid,strawcolouredfluid. Whichoneofthefollowingdiagnosisislikely?
A.Wilson'sdisease B.Haemochromatosis C.Pseudohypoparathyroidism D.Pseudopseudohypoparathyroidism E.Ankylosingspondylitis

Answer:b)haemochromatosis. The history of abdominal pain together with skin pigmentation and hepatomegaly suggest the patient has haemochromatosis. Haemochromatosis is associated with chondrocalcinosis, which commonly affects the 2nd and 3rd metacarpophalangeal joints.Theacutearthropathyislikelytobepyrophosphatecrystalarthritis.

Dr.KhalidYusufElzohrySohagTeachingHospital2012 10

www w.MRCPass.c com

Rheum matology

Rheumatol logyQ010 0
A 70 0 year old man presen nts with bo ony swelling gs of the DI IP joints on n both hand ds. They wer repainfulayearagobu utarenowpainless. Themostlikely ydiagnosisis i :
A.Heberdensnodes B.Bouch hardsnodes s C.Osler'snodes D.Gouty ytophi E.Charc cot'sjoints

Answer:a)Heb berden'sno odes. The emostlikely ydiagnosisi isosteoarth hritis.Atthe eDIPjoints, ,swellingar reknownas s Heb berdensnodesandatthe t PIPjoin nts,theyare eknownasBouchards snodes.

Heb berden'snode
Dr.Khalid K Yusuf fElzohrySo ohagTeachin ngHospital 2012 11

www.MRCPass.com

Rheumatology

RheumatologyQ011
A 60 year old man presents with gouty tophi. He has been commenced on allopurinol but develops severe joint pains 3 days later. On examination, he has a temperature of 39Canderythematousswellingofhiswrists,kneesandankles. Investigationsreveal:urate0.6(0.230.45),creactiveprotein180mg/L. Whichofthefollowingislikelytohavecausedthepresentation?
A.Allopurinol B.Colchicine C.Prednisolone D.Pseudogout E.Septicarthritis

Answer:a)allopurinol. Allopurinol blocks uric acid production and is the drug most often used in longterm treatment for older patients and overproducers of uric acid. Allopurinol is taken orally once a day in doses of 100 mg to 600 mg, depending on the patient's response to treatment. Between 3% to 5% of patients experience leukopenia, thrombocytopenia, diarrhea, headache,andfever.

Dr.KhalidYusufElzohrySohagTeachingHospital2012 12

www w.MRCPass.c com

Rheum matology

Rheumatol logyQ012 2
A 35 year old retired athlete presen nts with sev vere burnin ng pain affe ecting the right r leg and foot. 4 months earlier, he had several ar rthroscopic washouts for septic arthritis a affe ectinghisrig ghtknee,which w follow wedaninjury y. Examination re eveals a red duced range of movem ment of the e right knee e and ankle e. There sdiffuseswellingofthe erightlega andfootwit thoverlying gcool,scalyskin. was Wha atisthelike elydiagnosi is?
A.Comp partmentsyn ndrome B.Deepveinthromb bosis C.Reflex xsympatheticdystrophy y D.Rayna aud'sphenomenonaffec ctingtheleg E.Venou usvaricositie es

Answer:c)refle exsympath heticdystrophy. This s clinical sce enario is co ompatible w with a diagn nosis of refl lex sympathetic dystro ophy or com mplex regio onal pain syndrome s f following the previou us injury. Reflex R symp pathetic dyst trophy, also o known as s RSD, is a condition of burning pain, stiffn ness, swelling, and disc colorationof o theaffect tedarea. The e pain is often severe and a disprop portionate to t the signs s and follow w s a nonan natomic distribution.Th heskinchan ngesareduetotheass sociatedaut tonomicdy ysfunction.

Refl lexsympath heticdystro ophyaffectingtheright tfoot


Dr.Khalid K Yusuf fElzohrySo ohagTeachin ngHospital 2012 13

www.MRCPass.com

Rheumatology

RheumatologyQ012
A 75 year old has had increasing back and leg pains several years. X rays reveal bony sclerosis of sacroiliac, lower vertebral and upper tibial regions. He mentions greater difficultyhearingovertherecentyears. Bloodtestsrevealanelevatedserumalkalinephosphatase. Whatisthelikelydiagnosis?
A.Paget'sdiseaseofbone B.Osteoarthritis C.Osteomalacia D.Monoclonalgammapothyofuncertainsignificance E.Multiplemyeloma

Answer:a)Paget'sdiseaseofbone. In Paget's disease, onset of symptoms is usually insidious, with pain, stiffness, bone deformity,headaches,decreasingauditoryacuity,andincreasingskullsize. Signs may be bitemporal skull enlargement with frontal "bossing," dilated scalp veins, nerve deafness in one or both ears, angioid streaks in the fundus of the eye, and anterolateralbowingofthethighorlegwithwarmthandperiostealtenderness. Pagetic lesions are metabolically active and highly vascular and may lead to highoutput heartfailure. Deformities may develop from bow ing of the long bones or osteoarthritis of adjacent joints. Pathologic fractures may be the presenting finding. Characteristic xray findings include increased bone density, abnormal architecture, cortical thickening, bow ing, and overgrowth. Biochemistry includes elevated serum alkaline phosphatase (or bonespecific alkaline phosphatase) and increased urinary excretion of pyridinoline crosslinks. Serum calcium and phosphorus levels usually are normal, but serum calcium may increase during bed rest.

Dr.KhalidYusufElzohrySohagTeachingHospital2012 14

www w.MRCPass.c com

Rheum matology

Rheumatol logyQ013 3
A 55 year old patient pre esents with h a tw o week w history y of pain and stiffness s in her shoulders and wrists. The e symptoms s improve over o the day y. On exam mination, the ere was syno ovitisofbot thwristsan ndproximalmusclewasting.HerE ESRwas40mm/hr. atisthelike elydiagnosi is? Wha
A.Derm matomyositis B.Systemiclupuser rythematosus C.Rheum matoidarthr ritis D.Polym myalgiarheumatica E.Osteo oarthritis

Answer:c)rheumatoidart thritis. In a middle age ed female, acute arthr ritis of shou ulders and wrists w along g with synov vitis are high hlysuggestiveofacuteRhematoid dArthritis.

Synovitisinvolv vingthewri istinrheum matoidarthr ritis

Dr.Khalid K Yusuf fElzohrySo ohagTeachin ngHospital 2012 15

www.MRCPass.com

Rheumatology

RheumatologyQ014
A 45 year old woman presents with numbness and weakness of her upper and lower limbs.Shedevelopedasthmaattheageof30. On examination she looks unwell. She has palpable purpura over her face and over her elbowsandknees. On neurological examination she has a right sided wrist drop and there is weakness of dorsiflexion of her right foot. Sensation is also impaired over the dorsum of her right foot. Investigations: CXRisnormal. Hb10.9g/dL MCV90fl WBC23x10^9/l Eosinophils12% ANCAnegative Whatisthelikelydiagnosis?
A.Polyarteritisnodosa B.ChurgStrausssyndrome C.SLE D.Allergicbronchopulmonaryaspergillosis E.Takayasu'sarteritis

Answer:b)ChurgStrausssyndrome. Thecombinationofmononeuritismultiplex,asthma,eosinophilia. The American College of Rheumatology (ACR) has proposed 6 criteria for diagnosis of Churg Strauss syndrome. The presence of 4 or more criteria yields a sensitivity of 85% andaspecificityof99.7%. Thesecriteriaare (1)asthma(wheezing,expiratoryrhonchi) (2)eosinophiliaofmorethan10%inperipheralblood (3)paranasalsinusitis (4)pulmonaryinfiltrates(maybetransient) (5)histologicalproofofvasculitiswithextravasculareosinophils (6)mononeuritismultiplexorpolyneuropathy

Dr.KhalidYusufElzohrySohagTeachingHospital2012 16

www w.MRCPass.c com

Rheum matology

Vasculiticrashontheskin nofapatien ntwithChur rgStrausssy yndrome

Dr.Khalid K Yusuf fElzohrySo ohagTeachin ngHospital 2012 17

www.MRCPass.com

Rheumatology

RheumatologyQ015
A40year old manhasgeneralised jointpains andstiffness, particularlyin the knees. He alsohassore,dryeyesanddifficultytoleratingcontactlenses.Onexaminationthereare no joint swellings or effusions. His ESR is 80 mm/hour, Rheumatoid Factor positive with atitreof1/1024. Whichofthefollowingislikely?
A.PositiveantibodiestoRoandLaantigens B.PositiveantiSCL70antibody C.Positiveantimitochondrialantibodies D.PositiveantiSmantibodies E.PositiveANCA

Answer:a)positiveantibodiestoRoandLaantigens. Ro is also known as anti ssA and La is known as anti ssB antibody, both are diagnostic tests for Sjogrens. The history of dry eyes (keratoconjunctivitis sicca) and joint pains withstronglypositiveRhFgoeswithSjogrens.

Dr.KhalidYusufElzohrySohagTeachingHospital2012 18

www.MRCPass.com

Rheumatology

RheumatologyQ016
A 45 year old man has recurrent sinusitis and haemoptysis. He also has haematuria and mildrenalimpairment. TestsaresentforsuspectedWegener'sgranulomatosis. WhichoneofthefollowingtestshasgreatestspecificityforWegener'sgranulomatosis?
A.Antiglomerularbasementantibody B.PANCApositiveantibodiesproteinase3 C.PANCApositiveantibodiesmyeloperoxidase D.CANCApositiveantibodiesproteinase3 E.CANCApositiveantibodiesmyeloperoxidase

Answer:d)cANCApositiveantibodiesproteinase3. On immunofluoresecnce, if ANCA are present, the staining pattern may be cytoplasmic (cANCA) or perinuclear (pANCA). Typical antigen specificity includes c ANCA proteinase 3 which is more common in Wegener's granulomatosis. p ANCA myeloperoxidase is morecommoninpolyarteritisnodosa.

Dr.KhalidYusufElzohrySohagTeachingHospital2012 19

www.MRCPass.com

Rheumatology

RheumatologyQ017
A lady with w hiplash injury 5 years ago presents with pains in the neck and shoulder. Theywerenotrelievedby12cocodamolsaday. Whatshouldbedonenext?
A.Amitryptilline B.NSAIDs C.Physiotherapy D.Morphine E.Gabapentin

Answer:C)physiotherapy. Whiplash injuries and radiculopathies causing back pains can be difficult to treat with medications. Physiotherapy has an important role for symptom relief in combination withanalgesia.

Dr.KhalidYusufElzohrySohagTeachingHospital2012 20

www.MRCPass.com

Rheumatology

RheumatologyQ018
A 45 year old man presents with a week history of a painful right leg. He has had previous episodes of erythema nodosum, recurrent oral and scrotal ulceration. Examinationrevealsadiffuselyswollenleftleg. Whatisthelikelycauseofhisswollenleg?
A.Baker'scyst B.Gonococcalarthritis C.Reactivearthritis D.Deepveinthrombosis E.Cellulitis

Answer:d)Deepveinthrombosis. The overall diagnosis is Behcet's syndrome. There is a thrombotic tendency, hence the likelycauseofaDVT.

Dr.KhalidYusufElzohrySohagTeachingHospital2012 21

www w.MRCPass.c com

Rheum matology

Rheumatol logyQ019 9
A45 5 yearoldwoman w pres sentswithpain p in thewrist. The pain p iscent tred overth heradial stylo oidandisin ncreasedby yabductionofthethum mbagainstresistance. Wha atisthemo ostlikelydia agnosis?
A.Carpa altunnelsyndrome B.Rheumatoidarthr ritis C.DeQu uervainsten nosynovitis D.Osteo oarthritis E.Adduc ctorpolliciss synovitis

Answer:c)DeQuervain's Q tenosynovit tis. The epaininthe ethumbonresistedabductionistypical t ofDe eQuervain s. De Quervain's disease occurs o more e frequent tly in w om men. The age a group usually affe ectedis30to t 50yearolds. o The e history often is of unaccustome ed or excessive activity such as rose pruning. The pati ientcompla ainsofpainontheradi ialsideofth hewrist.Ab bductionofthethumbagainst resistance is painful. p Fink kelstein's te est is posit tive. This is s performed with the thumb flexedacrossth hepalmofthehand,asking a thepatient p tomove m thewr ristintoflex xionand ulna ardeviation n. This s stresses the t tendons s of abduct tor pollicis longus and d extensor pollicis bre evis and repr roducesthe epainofde eQuervain's stenosynov vitis.

Fink kelstein'ste est


Dr.Khalid K Yusuf fElzohrySo ohagTeachin ngHospital 2012 22

www.MRCPass.com

Rheumatology

RheumatologyQ020
A43yearrheumatoidarthritiswasinvestigatedroutinelyandhadthefollowingresults: Haemoglobin11.2g/dL Platelets385x10^9/L WhiteCellCount8.2x10^9/L MCV110fL Whichdrugisshelikelytohavebeenon?
A.Aspirin B.Rituximab C.Methotrexate D.Vincristine E.Hydroxychloroquine

Answer:C)Methotrexate Methotrexate is associated with bone marrow suppression, and can lead to pancytopenia or a megaloblastic anaemia, especially if folate treatment is not given as well.

Dr.KhalidYusufElzohrySohagTeachingHospital2012 23

www.MRCPass.com

Rheumatology

RheumatologyQ021
A75yearoldwomanhasrecentlybeencommencedonalendronateforosteoporosis. Whatisthemechanismofactionofalendronate?
A.Increasesosteoblastactivity B.Inhibitsosteoclastactivity C.IncreasesvitaminDabsorption D.Causeshypercalcaemia E.Increasestheactionofoestrogenonbone

Answer:b)Inhibitsosteoclastactivity. Alendronate is a bisphosphonate which can increase bone mineralisation by inhibiting osteoclasticactivity.

Dr.KhalidYusufElzohrySohagTeachingHospital2012 24

www.MRCPass.com

Rheumatology

RheumatologyQ022
An 75 year old man presents with bilaterally painful knees. He has bilateral reduced knee movements and crepitus. X ray shows sclerosis, osteophytes and loss of joint space. Whichoneofthefollowingisthemostappropriateinitialtreatment?
A.Ibuprofen B.Ibuprofenandlansoprazole C.Paracetamol D.Codeinephosphate E.Celecoxib

Answer:c)paracetamol. The principle goal of systemic therapy in osteoarthritis is to provide effective pain relief with least associated toxicity. Paracetamol is the recommended initial therapy, especiallyintheelderlyduetopossiblegastrointestinalupset.

Dr.KhalidYusufElzohrySohagTeachingHospital2012 25

www.MRCPass.com

Rheumatology

RheumatologyQ023
A35yearoldwomanrecentlyarrivedintheUKfromGhana.Shecomplainsofneckpain withpinsandneedlesaffectingtherightarmassociatedwithaweakgrip. Examination reveals tenderness over the cervical spine. X rays of the cervical spine shownarrowingoftheC3/4andC4/5jointspaceandpartialcollapseofC4. Investigationsshow: Hb9.5g/dl WCC11.1 platlets520 ESR120mminthefirsthour CRP250g/l Calcium2.21micromol/l Albumin32g/l alkalinephosphatase210units/l phosphate0.8micromol/l. Whatisthemostlikelydiagnosis?
A.Pottsdisease B.Osteoporosis C.Multiplemyeloma D.Ankylosingspondylitis E.Syringomyelia

Answer:a)Pottsdisease. Potts disease is tuberculous infection of the spine with associated collapse of the vetebral body. The infection spreads from tw o adjacent vertebrae into the adjoining disc space. If only one vertebra is affected, the disc is normal, but if tw o are involved theintervertebraldisc,whichisavascular,collapses. Signs and symptoms include: Localised back pain, Paravertebral swelling, Neurological signsincludingparaplegia. Drug treatment (antituberculous drugs) is generally sufficient for Potts disease, with spinal immobilisation if required. Surgery is required if there is spinal deformity or neurologicalsignsofspinalcordcompression.

Dr.KhalidYusufElzohrySohagTeachingHospital2012 26

www w.MRCPass.c com

Rheum matology

Pott tsdiseaseon o anMRI

Dr.Khalid K Yusuf fElzohrySo ohagTeachin ngHospital 2012 27

www w.MRCPass.c com

Rheum matology

Rheumatol logyQ024 4
A 35 3 year old woman presents w with tight skin over her hand ds with Raynauds phe enomenon. She has ulc ceration of the fingert tips and ass sociated sm mall white deposits. Shehasnotedincreasingb breathlessn nessoverth hepastfew years.Rena alfunctionis i mildly imp paired. atisthelike elydiagnosi is? Wha
A.SLE B.Ataxia atelangiecta asia C.Polym myositis D.Limite edcutaneou usscleroderm ma E.Diffus secutaneous sscleroderm ma

Answer:d)Lim mitedcutane eoussclerod derma. Limited cutan neous scleroderma is i also kn nown as CREST C synd drome (calcinosis, Ray ynauds,(o)e esophagealdysfunction,sclerodac ctyly,andte elangiectasia).Limited drefers to the t extent of skin inv volvement limited to the forearms and fac ce. They ge enerally dev velop pulmo onary hype ertension rather r than n pulmonar ry fibrosis, leading to ow ards brea athlessness s.

Tigh htthickened dskin(scler rodactyly)seeninScler roderma


Dr.Khalid K Yusuf fElzohrySo ohagTeachin ngHospital 2012 28

www w.MRCPass.c com

Rheum matology

Rheumatol logyQ025 5
A 70 7 year old d woman complains of o pain at the base of o her right thumb. There T is tend dernessand dswellingof o rightfirstcarpometa acarpaljoint t. Wha atisthelike elydiagnosi is?
A.DeQu uervain'sten nosynovitis B.Rheumatoidnodu ule C.Osteo oarthritis D.Psoria aticarthropa athy E.Reiter r'ssyndrome e

Answer:c)oste eoarthritis. Oste eoarthritisof o the1stcarpometac c arpaljointis i common. .Swellingis susuallybo onyhard due eosteophyteformation n.PIPjointnodesareknown k asBo ouchard'sand a DIPjoin ntnodes areknownasHeberden's H nodes.

Dr.Khalid K Yusuf fElzohrySo ohagTeachin ngHospital 2012 29

www.MRCPass.com

Rheumatology

RheumatologyQ026
A 70 year old patient has a set of investigations due to lethargy. She complains of polyuriaandgeneralisedbackpains.Uponinvestigation,shehasthefollowingresults: Hb8g/dl MCV100fl Platelets190x10^9/l totalprotein90g/l(6076)gm% Albumin35(3749)g/l calcium2.9(2.252.7)mmol/l phosphate0.75(0.88)pmol/l Whatistheappropriatenextinvestigation?
A.Completeliverfunctiontests B.Urinaryalbumin C.Plasmaelectrophoresis D.24hoururineprotein E.Uricacid

Answer:C)plasmaelectrophoresis. The patient has multiple myeloma as indicated by hypercalcaemia, polyuria and bone pains.

Dr.KhalidYusufElzohrySohagTeachingHospital2012 30

www.MRCPass.com

Rheumatology

RheumatologyQ027
A 72 year old lady has severe pain in the left knee and right hand. An X ray of the hand shows osteophytes and an X ray of the knee was normal. On examination, she has limitedlefthipflexion. Whatshouldbethenextinvestigationfortheknee?
A.MRIoftheknee B.Bonescan C.Arthroscopyoftheknee D.CTscanoftheknee E.Ultrasoundoftheknee

Answer:A)MRIoftheknee. The patients presentation suggests osteoarthritis, but an MRI would be helpful to exclude other pathology as well as confirm osteoarthritis in the context of a normal knee X ray. The MRI is useful for assessing the state of the cruciate ligaments and the joint cartilage as well as avascular necrosis. Arthroscopy is helpful but invasive in this instance.

Dr.KhalidYusufElzohrySohagTeachingHospital2012 31

www.MRCPass.com

Rheumatology

RheumatologyQ028
A 28 year old patient with systemic lupus erythematosus attends the obstetric clinic at 25 weeks into her pregnancy. The foetal heart rate is 45 beats per minute. Foetal echocardiographyshowscompleteheartblock. Whichoneofthefollowingmaternalautoantibodiesislikelytobepresent?
A.AntiRo(SSA) B.AntidsDNA C.AntiLa(SSB) D.AntiJo1 E.Anticentromere

Answer:a)AntiRo(SSA). AntiRo antibody is associated with congenital complete heart block. When congenital complete heart block occurs, SSA antibodies are almost alw ays present in maternal andfetalserum(maternalantiRo(SSA)antibodycrossestheplacenta).

Dr.KhalidYusufElzohrySohagTeachingHospital2012 32

www.MRCPass.com

Rheumatology

RheumatologyQ029
A 34 year old lady with systemic sclerosis complained of lethargy. Her blood pressure was185/90mmHg. Fundoscopyshowedcottonwoolspots.InvestigationsshowedthatherU+Eswere: sodium135mmol/l potassium4.5mmol/l urea12mmol/l creatinine225mol/l Whatisthetreatmentofchoiceforthispatient?
A.OralCaptopril B.IVProstacyclin C.IVLabetalol D.IVSodiumnitroprusside E.OralAtenolol

Answer:A)oralcaptopril. Amajorcomplicationofsclerodermaisrenalcrisis.Thisischaracterisedbyabruptonset of severe hypertension, usually retinopathy, together with rapid deterioration of renal functionandheartfailure. Hypertension should be treated with an ACE inhibitor. This is because the underlying pathology causing hypertension is angiotensin IIinduced vasoconstriction, and trials have shown ACE inhibitors to have the best antihypertensive efficacy and improved survival. The aim is to reduce pressure gradually, as an abrupt fall can lead to cerebral ischemia /infarctions (as in any accelerated hypertension), and may cause decreased renal perfusion as well as acute tubular necrosis. Calcium channel blockers may be added to ACE inhibitors. Parenteral antihypertensive agents (such as intravenous nitroprusside or labetalol)shouldbeavoidedastheycauseabruptbloodpressuredrops.
Dr.KhalidYusufElzohrySohagTeachingHospital2012 33

www w.MRCPass.c com

Rheum matology

Rheumatol logyQ030 0
A 45 year wom man presen nts with an 8 month history h of jo oint pains and stiffness s of the han ndsandfeet t. Examination re eveals a sy ynovitis of t the distal interphalan i ngeal joints, , left index x finger, righ htwristandanklejoints. Nail lpittingwas snoticed.Her H ESRwas s20mm/hr. ichoneoffollowing fo co onditionsisassociated a withthepa atternofjoin ntinvolvem ment? Whi
A.SLE B.Psoria asis C.Rheum matoidarthr ritis D.Septicarthritis E.React tivearthritis

Answer:b)pso oriasis. The e diagnosis is psoriat tic arthritis s. Psoriatic arthritis is i subclassi ified according to diffe erent patte erns of ar rthritis: asy ymmetrical oligoarthritis, symmetric polya arthritis, spondyloarthro opathy and arthritis mutilans. In about a 20% of patients there is a chronic, c prog gressive, deforming d arthropath hy in an asymmetric a cal pattern n, including g distal inte erphalangea aljointinvolvement.

PsoriaticArthro opathyno oteonycholy ysisonthenails


Dr.Khalid K Yusuf fElzohrySo ohagTeachin ngHospital 2012 34

www w.MRCPass.c com

Rheum matology

Rheumatol logyQ031 1
A 16 year old girl presents with wid despread pa alpable pur rpura over legs and buttocks. Shehasabdom minalpainsand a isnoted dtohavebl loodandpr roteinonur rinedipstick k. Wha atisthemo ostlikelydia agnosis?
A.Juven niledermatomyositis B.Juven nilechronica arthritis C.HenochSchnlein npurpura D.Rheumatoidarthritis E.HUSTTP T

Answer:c)Hen nochSchnl leinpurpura a. Hen nochSchnlein purpur ra (HSP) is a systemic vasculitis mostly m seen n in children n. It is a mul ltisystemdisorderinvo olvingthesk kin,joints,gastrointest g tinalandren naltracts. Aetiologyisunknown,but tthesyndro omeisoften nprecededbyinfectionssuchasGroup G A beta a hemolyt tic streptococcal res spiratory tract t infect tion, Campylobacter jejuni, Myc coplasma pneumoniae p e and viruse es such as varicella, hepatitis h B, EpsteinBarr virus, andparvovirus sB19. Path hologyisdu uetointravasculardep positionofIgAimmune ecomplexes swithactivationof com mplement and a leucocy yte infiltrat tion. Patien nts often present p wit th a purpuric rash usually involving the but ttocks and lower limb bs, arthralg gia and join nt swelling, severe colic cky abdominal pain an nd tenderness caused by vasculit tisinduced thrombosis in the gut. . Ren nal involvem ment comm monly prese ents as mic croscopic ha aematuria and proteinaemia. The emostserio ouslongterm

com mplicationfr romHSPisprogressive erenalfailure.

Dr.Khalid K Yusuf fElzohrySo ohagTeachin ngHospital 2012 35

www w.MRCPass.c com

Rheum matology

Rheumatol logyQ032 2
A 45 5 year old man m has dif fficulty ben nding touching his toes s w hen ben nding over. . He has lower back pain, stiffness in the thor racic region and reduce ed chest ex xpansion. His lower ne Xray is s shown ab bove. He has h negativ ve rheuma atoid factor r, and a HLA H B27 spin genotype. atisthedia agnosis? Wha
A.Paget t'sdisease B.Osteo oarthritis C.Marblebonedisease D.Marfa an'ssyndrom me E.Ankylosingspondylitis

Answer:e)ank kylosingspo ondylitis. Ank kylosing spo ondylitis is a seronegative arthrop pathy assoc ciated with HLAB27 ge enotype. It le eads to inc creased stif ffness and fusion of the t vertebr rae causing g a bamboo o spine. Typically,thesacroiliacjoi intsandhip pjointsareaffected,bu utthoracicandcervica alspines areeventuallyaffectedas swell. The ere is a list of associat tions mostl ly starting with w A. The ese are ant terior uveit tis/iritis, aort tic regurgita ation, aortic aneurysm m, apical fibrosis, asper rgillosis infe ection, amy yloidosis andarchilleste endinitis. In bamboo b spine, there is s fusion of the t vertebr ral bodies and squaring g of the ver rtebrae. One e also expe ects syndesm mophytes (thin ( vertical dense sp picules brid dging the ve ertebral bod dies), Romano lesions (erosion at t the corner r of vertebral bodies) and enthes sopathy (liga amentorte endoncalcif fication).

Bam mbooSpine
Dr.Khalid K Yusuf fElzohrySo ohagTeachin ngHospital 2012 36

www w.MRCPass.c com

Rheum matology

Rheumatol logyQ033 3
A 35 5 year old woman w pre esents with red scaly plaques p on her cheeks and her fo orehead. Oncloser c exam mination,therewasplu uggingofso omehairfolliclesandatrophyofth heskin. Wha atisthelike elydiagnosi is?
A.Druginducedlupus B.Psoria asis C.Discoidlupus D.Sarco oidosis E.Lupus spernio

Answer:c)disc coidlupus. The e diagnosis is discoid lupus ery ythematosu us. Lesions are discre ete plaques s, often eryt thematous, scaly, with h extension n into hair follicles. f Th hese lesions s can occur r on the face e, scalp, in the pinnae e, behind t the ears or on the ne eck. There can also be e active indu uratederyth hemaandc centralatrophicscarrin ng.

Disc coidlupus
Dr.Khalid K Yusuf fElzohrySo ohagTeachin ngHospital 2012 37

www.MRCPass.com

Rheumatology

RheumatologyQ034
A 50 year old patient with active rheumatoid arthritis has failed treatment with gold, methotrexate, hydroxychloroquine and sulphasalasine. She has ongoing joint pains and erosivedamage. Whatisthemostappropriatetreatment?
A.Highdoseprednisolone B.COX2inhibitors C.Infliximab D.Ciclosporin E.Azathioprine

Answer:c)infliximab. The AntiTNF drugs are infliximab and adalimumab. The criteria for treatment with these drugs are that patients who have been treated with at least tw o DMARDs (diseasemodifyingdrugs)continuetohaveactiverheumatoidarthritis. Examples of the DMARDs are: gold injections, sulphasalazine, hydroxychloroquine, leflunomide, cyclosporin, azathioprine and methotrexate [One of the tw o must be methotrexate].

Dr.KhalidYusufElzohrySohagTeachingHospital2012 38

www.MRCPass.com

Rheumatology

RheumatologyQ035
A 45 year old woman presents with a year's history of Raynauds phenomenon, dyspepsia and joint pains. On examination, she has sclerodactyly and synovitis of small joints of her hands. Her ESR is 60 mm/hr (<10) but Rheumatoid factor and Antinuclear Antibodyarebothnegative. Whatotherclinicalfeatureislikelyinthispatient?
A.Splinterhaemorrhages B.Erythemamarginatum C.Butterflyrash D.Anterioruveitis E.Smallbowelhypomotility

Answer:e)smallbowelhypomotility. This woman features of a mixed connective tissue disorder such as CREST/systemic sclerosis, sclerodactyly, Raynauds, dyspepsia and arthralgia. The other likely developmentwouldbemalabsorptionwhichisassociatedwithhypomotilityofthesmall bowel.

Dr.KhalidYusufElzohrySohagTeachingHospital2012 39

www.MRCPass.com

Rheumatology

RheumatologyQ036
A 60 year old man is on treatment for chronic heart failure with diuretics. He has preivous history of athma. He presents with sudden onset of pain and swelling of the metatarsophalangeal joint of his right big toe. Aspiration of the joint demonstrates crystalsofmonosodiumurate. Whatistherecommendedtreatment?
A.Aspirin B.Colchicine C.Nonsteroidalantiinflammatorydrugs D.Cyclooxygenase2inhibitor E.Allopurinol

Answer:b)Colchicine. Inthisparticularpatient,colchicineisthebestoption.Inacutegout,eithercolchicineor NSAIDscanbeused.However,asthmacontraindicatesNSAIDs.

Dr.KhalidYusufElzohrySohagTeachingHospital2012 40

www w.MRCPass.c com

Rheum matology

Rheumatol logyQ037 7
A 35 5 year old man m is reno ovating his apartment w hen he slams s a doo or against his h foot. He develops sw welling, ery ythema and d pain in all l the digits of his foot. He has an n ESR of mm/houran ndatemper ratureof36 6C. 20m atisthelike elydiagnosi is? Wha
A.Porph hyria B.Cellul litis C.Gout D.Rayna aud'sphenomenon E.Reflex xsympatheticdystrophy y

Answer:e)reflexsympath heticdystrophy. Refl lex sympat thetic dystr rophy occur rs following g trauma to o an injured d part of th he body, and can progr ress to other parts. It t is due to autonomic c nervous system s dys sfuction. Sym mptomsofextreme e painandburningcanoccur.Analgesicsareoftenunhelpful.

Flus shing,shinyandatroph hiedskinon ntheleftleg gindicatingreflexsymp patheticdystrophy

Dr.Khalid K Yusuf fElzohrySo ohagTeachin ngHospital 2012 41

www.MRCPass.com

Rheumatology

RheumatologyQ038
A 42 year old lady has tightening of the skin around her hands and mouth. She has several telangiectasia on her hands, and complains of severe cold hands in w inter. She wasnoticedtobepale.Investigationsshow: Hb4.5g/dl MCV105fl WCC6x10^9/l platelets230x10^9/l Iron22(1429)mol/l Ferritin155(15200)mol/l totalironbindingcapacity50(4572)umol/l Folate11(320)g/l VitaminB1285(120700pmol/l) Whatisthelikelycauseofanaemia?
A.Perniciousanaemia B.Secondaryfolatedeficiency C.Celiacdisease D.Methotrexate E.Bacterialovergrowth

Answer:E)bacterialovergrowth. Scleroderma can cause folate deficiency due to malabsorption. How ever, in this case, thefolatelevelsarenormalandthereisB12deficiency. B12 deficiency can occur in conditions w here there is bacteria overgrowth in the small intestine (blind loop syndromes) such as jejunal diverticulosis, Crohns disease, fistulas and scleroderma. The anaerobic organisms metabolise vitamin B12 and impair absorption.Whensystemicsclerosis(SSc)involvesthesmallintestine,normalperistaltic movementsarelostandmotilityisimpairedleadingtostasisanddilatation.

Dr.KhalidYusufElzohrySohagTeachingHospital2012 42

www w.MRCPass.c com

Rheum matology

Rheumatol logyQ039 9
A55 5yearoldman m develop pssuddeno onsetseverepaininhis srightbigtoe. Onexaminationhehassw wellingofth hemetacarp pophalange ealjointofhis h righthallux.The round skin is i erythema atous. It is t tender to to ouch. Initial investigations reveal araised surr whitecell,coun ntandanelevatedCRP P. atshouldbe edonetoco onfirmthediagnosis? d Wha
A.MRIof o thetoe B.Serum muricacid C.Serum mrheumatoid D.Jointfluidaspirateformicroscopy E.Serum mpyrophosp phatelevels

Answer:d)join ntfluidaspirateformic croscopy. The e likely diagnosis is acu ute gout. Th he serum in nflammatory y markers are a raised, but uric acid dlevelsmay ybenormal l. Join nt fluid asp pirate best test. Polar rised light microscopy y shows str rongly biref fringent (neg gativesign)needlesha apedcrystals.

AcuteGoutinv volvingtheleft l foot


Dr.Khalid K Yusuf fElzohrySo ohagTeachin ngHospital 2012 43

www.MRCPass.com

Rheumatology

RheumatologyQ040
A 80 year old man developed acute monoarthritis of his right ankle following an admission with congestive cardiac failure. He mentioned that he had ankle oedema and hadrecentlybeenprescribedfrusemidebytheGP. Whatisthelikelydiagnosis?
A.Rheumatoidarthritis B.Gout C.Pseudogout D.Osteoarthritis E.Septicarthritis

Answer:b)Gout. Goutcanbeprecipitatedbydiuretics,e.g.frusemide.

Dr.KhalidYusufElzohrySohagTeachingHospital2012 44

www.MRCPass.com

Rheumatology

RheumatologyQ041
A 45 year old has a past history of systemic sclerosis. She now has headaches and blurredvision.Onexamination,shehasabloodpressureof220/100mmHgandthereis evidenceofbilateralpapilloedema. Whichofthefollowingmedicationsshouldbeprescribed?
A.Oralhydrochlorothiazide B.OralLisinopril C.SublingualNimodipine D.IVSodiumNitroprusside E.IVLabetolol

Answer:b)OralLisinopril. A major complication of scleroderma is renal crisis which is characterised by abrupt onsetofseverehypertension.Thehypertensionalmostalwaysisseverewithadiastolic BP over 100 mmHg in 90% of patients. There is associated hypertensive retinopathy in about 85% of patients. ACE inhibitors are first line, with an aim to reduce the blood pressuregradually.

Dr.KhalidYusufElzohrySohagTeachingHospital2012 45

www.MRCPass.com

Rheumatology

RheumatologyQ042
A 50 year old man presents an acute monoarthritis of left knee. Gout is confirmed following joint aspiration and examination of fluid under polarised light microscopy. He had also underwent endoscopy 3 weeks earlier because of indigestion this confirmed a duodenalulcer. Whichoneofthefollowingisthebesttreatmentforthepatient?
A.Allopurinol B.Intraarticularcorticosteroidinjection C.IndomethacinandLansoprazole D.CelecoxibandLansoprazole E.IndomethacinandMisoprostol

Answer:b)Intraarticularcorticosteroidinjection. All nonsteroidals including Cox II selective inhibitors are relatively contraindicated in the presence of active ulceration. In a large joint such as the knee, the safest option wouldbeinjectcorticosteroidintothejoint.

Dr.KhalidYusufElzohrySohagTeachingHospital2012 46

www.MRCPass.com

Rheumatology

RheumatologyQ043
A 62 year old man has a 5 week history of pain and swelling affecting left knee, both anklesandhisrightwrist. Hehaslost6kginweight. Hisinvestigationsshow: WCC14.1X10^9/l Hb10.3X10^9/l MCV72fl plt510X10^9/l ESR63mminthefirsthour CRP21g/l CK120iu RhF1/80 ANAnegative ENAnegative XRofhandsandfeetnormal. Whatisthemostlikelydiagnosis?
A.Paraneoplasticsyndrome B.Osteoarthritis C.Polymyositis D.Scleroderma E.Behcet'ssyndrome

Answer:a)Paraneoplasticsyndrome. The CK is not significantly raised. Weight loss and anaemia suggest underlying malignancy. Paraneoplastic syndrome can present with an assymetrical arthralgia which more commonly affects the lower limbs. False positive rheumatoid factor can occur but shouldbeoflowtitre.

Dr.KhalidYusufElzohrySohagTeachingHospital2012 47

www w.MRCPass.c com

Rheum matology

Rheumatol logyQ044 4
A 45 year old lady has h had long sta anding arth hritis of her r hands. He er has hand d Xrays don ne. Whi ich of the following X ray cha anges suggests rheum matoid arth hritis instea ad of a sero onegativearthropathy a ?
A.Osteo osclerosis B.Osteo ophytes C.Osteo oporoticchan nges D.Periarticularerosions E.Lossof o jointspace e

Answer:d)per riarticularer rosions. Oste eophytesan ndlossofjo ointspaceare a common nlyfoundin nosteoarthritis,althou ughthey can also be fou und in rheu umatoid art thritis. Peria articular ero osions are most m sugge estive of rheu umatoidart thritis.

Peri iarticularer rosions


Dr.Khalid K Yusuf fElzohrySo ohagTeachin ngHospital 2012 48

www.MRCPass.com

Rheumatology

RheumatologyQ045
A 76 year old woman presents with weakness of his hand. On examination there was tenderness, crepitus and bony swellings over the base of the first metacarpal and wastingoftherightthenareminence. InvestigationsrevealanESRof25mm/1sthr,aCRPof10mg/L,aUrateconcentrationof 0.42 (0.190.36). Her Rheumatoid factor was 60 IU/L (<30). An xray of the right hand showed a loss of joint space, periarticular sclerosis and osteophytes of the first carpometacarpaljoint. Whatisthelikelydiagnosis?
A.Osteoarthritis B.DeQuervainstenosynovitis C.Pseudogout D.Gout E.Rheumatoidarthritis

Answer:a)osteoarthritis. The patient has clinical and radiological features consistent with osteoarthritis (OA) of the 1st right carpometacarpal (CMC) joint. The condition is characterised by joint pain, crepitus and stiffness after movement. Joint swellings are bony in nature (Bouchard's and Heberden's nodes), unlike boggy swellings which occurs in inflammatory arthritis. This patients inflammatory markers are mildly raised only, making an inflammatory arthritisunlikely.

Dr.KhalidYusufElzohrySohagTeachingHospital2012 49

www w.MRCPass.c com

Rheum matology

Rheumatol logyQ046 6
A 70 0 year old man m presen nts with sev vere back pains. An His s total serum protein is i 85 g/l with hanalbumi inof41g/l. A chest X ray y shows sev veral lucen ncies in ver rtebral bod dies. A ster rnal bone marrow aspi irateobtain nsadarkred djellylikem materialinthesyringe. a islik kelytoshow wwhichpro ominentfea atures? Thesmearofaspirate
A.Macrophages B.Osteo oblasts C.Plasm macells D.Pneumocytes E.Osteo osarcoma

Answer:c)Plas smacells. The e diagnosis is multiple myeloma. The bone marrow ne eedle is like ely to be in n a lytic lesio on filled with w plasma a cells. His s serum glo obulins are e high from m the mon noclonal gam mmopathy.

Apl lasmacell

Dr.Khalid K Yusuf fElzohrySo ohagTeachin ngHospital 2012 50

www.MRCPass.com

Rheumatology

RheumatologyQ047
A75yearoldmanpresentswithbilateralhippains.Investigationsreveal: Correctedcalcium2.5(2.22.6)mmol/l ESR22mm/1sthr Alkalinephosphatase800iu/L(50100) gammaGT22iu/L Whatisthelikelydiagnosis?
A.Osteoporosis B.Osteomalacia C.Paget'sdiseaseofthepelvis D.Polymyalgiarheumatica E.Multiplemyeloma

Answer:c)Paget'sdiseaseofthepelvis. This elderly patient presenting with bone pains has significantly elevated alkaline phosphatasebutnormalcalciumconcentrationssuggestingadiagnosisofPaget's.

Dr.KhalidYusufElzohrySohagTeachingHospital2012 51

www w.MRCPass.c com

Rheum matology

Rheumatol logyQ048 8
A 25 year old Turkish ma an was note ed by opht thalmologist ts to have a posterior r uveitis requ uiringhighdoseimmunosuppress sion.Hehas sahistoryof o recurrent tmouthulc cersand pain nfululcerson o thescrot tum. atisthelike elydiagnosi is? Wha
A.Behce et'ssyndrom me B.Reiterssyndrome e C.Ankylosingspondylitis D.Takay yasu'sarterit tis E.Giantcellarteritis s

Answer:a)Beh hcet'ssyndr rome. Beh hcet's syndrome is cla assically ch haracterized d as a triad of symptoms that include recu urring crops s of mouth ulcers (called apthous ulcers), genital ulcer rs, and uvei itis. The ulce ers are usua ally painful. The diseas se is more frequent f an nd severe in n patients fr rom the East ternMedite erraneanan ndAsiathan nthoseofEuropean E de escent. Eye manifestat tions may result r in bli indness. In addition, iritis, retina al vessel occ clusions and optic neur ritis can be found. Hyp popyon uve eitis (pus in n the anterior chamber of the eye), which is i consider red the ha allmark of f Behets disease, is i in fact a rare man nifestation. The arthrit tis of Behe ets disease e is usually intermitten nt, selflimit ted, not defo ormingandlocalizedto otheknees sandankles s.

Oralulceration ninBehcet's sdisease

Dr.Khalid K Yusuf fElzohrySo ohagTeachin ngHospital 2012 52

www.MRCPass.com

Rheumatology

RheumatologyQ049
A 70 year old woman has polydipsia and polyuria for 3 months. She also complains of loinpains.Shehasnotbeenonanymedication.Investigationsreveal: serumurea6(2.57.5)mol/l serumcreatinine80(60110)mol/l serumalbumin38g/L(3749) serumtotalcalcium3.1(2.22.6) Whatisthelikelycauseofthehypercalcaemia?
A.Multiplemyeloma B.Sarcoidosis C.Paget'sdisease D.Primaryhyperparathyroidism E.VitaminDtoxicity

Answer:d)Primaryhyperparathyroidism. PrimaryhyperparathyroidismiscausedbyanoverproductionofPTH. Excess PTH results in an increase in bone breakdown by means of osteoclastic resorption with subsequent fibrous replacement and reactive osteoblastic activity. Historically, in classic primary hyperparathyroidism, nephrolithiasis was noted in 50% of patients, and it was the most common clinical presentation of the disease. Additional manifestations of primary hyperparathyroidism include pancreatitis, peptic ulcer disease,andcardiovascularabnormalities.

Dr.KhalidYusufElzohrySohagTeachingHospital2012 53

www.MRCPass.com

Rheumatology

RheumatologyQ050
A42yearoldladypresentswithbackache.Herbloodresultsareasfollows: Hb11g/dL Ca1.9mmol/L Phosphate0.8mmol/L Alkalinephosphatase220U/L Albumin38g/L Urea7mmol/L Sodium142mmol/L Potassium3.9mmol/L Whatisthediagnosis?
A.Osteoporosis B.Paget'sdisease C.Osteomalacia D.Xlinkedhypophosphataemicrickets E.Hypoparathyoroidism

Answer:c)osteomalacia. The patient has osteomalacia with secondary hyperparathyroidism causing low phosphate levels. Osteomalacia is due to vitamin D deficiency which could be due to malabsorptionordietarydefiency,orrenal/liverdisease.

Dr.KhalidYusufElzohrySohagTeachingHospital2012 54

www.MRCPass.com

Rheumatology

RheumatologyQ051
A 32 year old woman has a deep vein thrombosis. Her previous history included investigationsformiscarriages. Investigations revealed: Haemoglobin 11.9 g/dl, White cell count 4 x 10^9/L, Platelet count30x10^9/L. Whichofthesearelikelytobeabnormal?
A.Homocysteinelevel B.ANCA C.ProteinC D.Antiphospholipidantibody E.Coomb'stest

Answer:d)Antiphospholipidantibody. Antiphospholipid syndrome leads to venous and arterial thrombosis, livedo reticularis, splinter hemorrhages, leg ulcer, multiinfarct dementia, chorea, Thrmobocytopenia (40%ofpatients),hemolyticanemiaandlatetermmiscarriages.

Dr.KhalidYusufElzohrySohagTeachingHospital2012 55

www.MRCPass.com

Rheumatology

RheumatologyQ052
A 35 year old man presents with acute stiffness swelling of his knees and ankles, a painful rash on his legs. The ESR was 100 mm/hour. Chest Xray showed hilar lymphadenopathybilaterally. Whatisthelikelyprogressionofthearthralgia?
A.Chronicarthritis B.Septicarthritis C.Improvementonlywithsteroids D.Spontaneousimprovement E.Permanentjointdestruction

Answer:d)spontaneousimprovement. The description typical of acute sarcoidosis erythema nodosum, oligoarthropathy and hilar lymphadenopathy. This usually has a good prognosis, with resolution over 68 weeks.

Dr.KhalidYusufElzohrySohagTeachingHospital2012 56

www.MRCPass.com

Rheumatology

RheumatologyQ053
A 75 year old lady presents with back pains. Lateral spine Xrays and pelvic xrays show osteopenia. A serum corrected calcium is 1.8 mmol/l and phosphate is 0.6 mmol/l. Alkalinephosphataseis360U/l. Whichdiagnosisismostlikely?
A.Myeloma B.Osteoporosis C.Osteomalacia D.Paget'sdisease E.Ankylosingspondylitis

Answer:c)osteomalacia. Osteomalacia is more likely than osteoporosis due to the low calcium, low phosphate andraisedalkalinephosphatse.TheconditioniscausedbylowvitaminDlevels. Alkaline phosphatase is raised w hen there is increased osteoblastic activity, conditions itmayberaisedin: Paget'sdisease osteomalaciaandrickets renalosteodystrophy bonemetastases

Dr.KhalidYusufElzohrySohagTeachingHospital2012 57

www.MRCPass.com

Rheumatology

RheumatologyQ054
A 22 year old lady has a sw inging fever, half a stone weight loss over 2 months, generalised myalgia, polyarthralgia affecting wrists, knees, ankles, elbows metacarpophalangealjointsandasorethroat. Investigations show : Hb 9.5g/l, MCV 85 fl, ESR 92 mm in first hour, CRP 45 g/l, serum ferritin1600mg/dl,RFnegative,ANAnegative,ENAnegative,ASOtitre<200iu. Whatisthelikelydiagnosis?
A.Rheumatoidarthritis B.Ankylosingspondylitis C.AdultonsetStill'sdisease D.Inclusionbodymyositis E.Polymyositis

Answer:c)AdultonsetStill'sdisease. Adult Onset Stills Disease (AOSD) is an acute febrile illness in young adults. It usually affectsmultipleorgans,butisadiagnosisofexclusion. Clinical features include a high fever, arthralgia and arthritis, phayngitis, typical rash (evanescent salmoncolored, macular or maculopapular eruption), lymphadenopathy, andserositis.Chronicarthritisandconstitutionalsymptomsarecommon. The triad of fever, rash, and arthralgia are often absent during the first month of the illness.Theusualjointsaffectedarewrists,knees,andanklesindescendingorder. Tw o thirds of cases experience polyarticular arthritis and one third have monoarticular symptoms. Approximately 1/3 of patients have chronic persistent disease with progressive joint damage.

Dr.KhalidYusufElzohrySohagTeachingHospital2012 58

www.MRCPass.com

Rheumatology

RheumatologyQ055
A65yearoldwomanhasaswollen,erythematouskneejoint.Aspirationofthejointwas performed.Microscopyshowedpositivelybirefringentcrystals. Whatarethecrystalscomposedof?
A.Calciumcarbonate B.Magnesiumsulphate C.Urate D.Calciumpyrophosphate E.Aminolaevulinicacid

Answer:D)calciumpyrophosphate. Pseudogout is caused by the deposition of calcium pyrophosphate deposition rather than the deposition of uric acid derivatives that cause gout. Also, in pseudogout, synovial fluid samples obtained with aspiration have positive birefringence. This finding isindirectcontrasttothenegativebirefringenceingout.

Dr.KhalidYusufElzohrySohagTeachingHospital2012 59

www.MRCPass.com

Rheumatology

RheumatologyQ056
A45yearoldwomanpresentswithclaudicationinherlowerlimbs.Sheisnotedtohave absentleftarmpulses. Thepreviousyearshehadasmallhemisphericcerebrovascularinfarct. Whatisthelikelydiagnosis?
A.Giantcellarteritis B.Takayasu'sarteritis C.Familialhypercholesterolaemia D.Coarctationoftheaorta E.Antiphospholipidsyndrome

Answer:b)Takayasu'sarteritis. Takayasu's arteritis is a large vessel vasculitis of unknown origin. The vasculitic process involves structures such as the aorta, great vessels, the sclera and the cardiac conductiontissues. Women are affected more than men, usually in the second and third decades of life. Presentation is often with symptoms such as fever, weight loss, night sweats and arthralgias. Symptoms related to ischaemia may include ischaemic stroke, visual disturbancesandclaudication.

Dr.KhalidYusufElzohrySohagTeachingHospital2012 60

www.MRCPass.com

Rheumatology

RheumatologyQ057
A 70 year old man develops weakness of the shoulders and around the hips over a 6 month period. He has also noticed weak finger flexors. He complained of difficulty sw allow ing liquids. There is no other significant past medical history. He smokes 15 cigarettes a day and drinks a bottle of w ine at the weekend. A creatinine kinase level comesbackat120U/l. A muscle biopsy sample shows myopathic changes. There are also inflammatory infiltratesandintracytoplasmicvacuolespresent. Whatisthelikelydiagnosis?
A.Polymyositis B.Fibromyalgia C.Polymyalgiarheumatica D.Dermatomyositis E.Inclusionbodymyositis

Answer:e)inclusionbodymyositis. The diagnosis is inclusion body myositis (IBM). This is an inflammatory condition affecting patients over the age of 50. Proximal muscles in the upper and lower limbs, and finger flexors are predominantly involved. The onset of muscle weakness in IBM is generally gradual (over months or years).Dysphagia is common, occurring in 4066% of patients. IBM occurs more frequently in men than w omen. CK may be normal. A muscle biopsy usuallyshowsintracytoplasmicinclusionsandalsoinflammatoryinfiltrates.

Dr.KhalidYusufElzohrySohagTeachingHospital2012 61

www w.MRCPass.c com

Rheum matology

Thig ghatrophyseen s ininclusionbodymyositis

Inclusionbodie esandinflam mmatoryin nfiltrates(ar rrows)

Dr.Khalid K Yusuf fElzohrySo ohagTeachin ngHospital 2012 62

www w.MRCPass.c com

Rheum matology

Rheumatol logyQ058 8
A 53 5 year old d woman with w rheum matoid arth hritis was r referred with iron de eficiency anaemia. She had exten nsive invest tigations. Endoscopy E showed ga astritis and d antral sions, small bow el biopsy sho owing mild villous blu unting, and d apopotic bodies, eros Colo onoscopywas w normal. atisthecau useofthese efindings? Wha
A.Meth hotrexate B.Azath hioprinether rapy C.Crohn nsdisease D.NSAID Dtherapy E.Felty'ssyndrome

Answer:d)NSA AIDtherapy y. The eendoscopy yfindingsar resuggestiv veofNSAID Drelatedgas stritis,whic chisthemo ostlikely causeofirondeficiencyan naemia.

NSA AIDrelatedgastritis
Dr.Khalid K Yusuf fElzohrySo ohagTeachin ngHospital 2012 63

www.MRCPass.com

Rheumatology

RheumatologyQ059
A 45 year old woman complains of 12 months of mid and lower back pain associated withstiffnessinhershoulders,wrists,smalljointsofthehands,hipskneesandfeet.The joint stiffness is more pronounced in the morning and lasts 23 hours and is associated withseverefatigue. She also experiences occasional pins and needles affecting all of the right hand and frequentfrontalandtemporalheadache. Clinical examination demonstrates no synovitis but tenderness around the base of the cervical spine, across the shoulders, over the costochondral cartilages, greater trochanterandtheknee. Investigationsareasfollows:CRP<5g/l,ESR20mminthefirsthour,CK120U/l,serum immunoglobulinsandproteinelectrophoresisnormal. Whatisthediagnosis?
A.SLE B.Polymyalgiarheumatica C.Fibromyalgia D.Dermatomyositis E.Polymyositis

Answer:c)Fibromyalgia. The above symptoms are all characteristic for fibromyalgia. The inflammatory markers andCKarenotraised,makingtheotherdiagnosesunlikely

Dr.KhalidYusufElzohrySohagTeachingHospital2012 64

www.MRCPass.com

Rheumatology

RheumatologyQ060
A 35 year old man presents with hot swollen elbow joint. He has a temperature of 37C andCRPof350mg/l.Adiagnosisofsepticarthritisissuspected. Whichofthefollowingorganismsismostcommonlyisolatedfromjoints?
A.Staphyloccocusaureus B.Neisseriagonorrhoeae C.haemolyticstreptococcus D.Pseudomonasaeruginosa E.Moraxellacatarrhalis

Answer:a)staphyloccocusaureus. Staph aureus is the commonest organism (over 50%). Neisseria gonorrhea can occur in patients with sexually transmitted disease. haemolytic streptococci can cause impetigo, sore throat and rheumatic fever. Moraxella is a gram negative coccus which cancausepneumoniainCOPDpatients.

Dr.KhalidYusufElzohrySohagTeachingHospital2012 65

www.MRCPass.com

Rheumatology

RheumatologyQ061
A 66 year old lady had a fall and fractured femur. A DEXA scan shows reduced bone mineraldensityconsistentwithosteoporosis. Whattreatmentshouldbegiventoreducethelikelihoodoffuturefractures?
A.Alendronate B.Alendronateandcalcium C.Alendronate,calciumandvitaminD D.Calcium E.VitaminD

Answer:C)alendronate,calciumandvitaminD Any patient above the age of 65 with osteoporosis is recommended to be on bisphosphonates. In addition, this patient has sustained a fracture, and should also be oncalciumandvitaminD(calcichewD3)aswell.

Dr.KhalidYusufElzohrySohagTeachingHospital2012 66

www.MRCPass.com

Rheumatology

RheumatologyQ062
A82yearoldwomanpresentswithconfusion.OfnoteinherpasthistorywasRaynauds phenomenon. Herinvestigationsshow: haemoglobin9.7g/dl whitecellcount3.6x10^9/l plateletcount100x10^9/l serumtotalprotein120g/l serumimmunoglobulins:IgA0.75g/l(0.83),IgG16g/l(613),IgM35g/l(0.42.5) Whichofthefollowingcomplicationsisshelikelydevelop?
A.Urinarytractinfection B.Hyperviscositysyndrome C.Pathologicalbonefracture D.Acuterenalfailure E.Erythemanodosum

Answer:b)Hyperviscositysyndrome. ThelikelydiagnosisisWaldenstromsMacroglobulinaemia(WM). Waldenstroms macroglobulinaemia is a type of nonHodgkins lymphoma. It is a condition which typically presents in the seventh and eighth decade of life. It is characterized by the presence of a high level of a macroglobulin immunoglobulin M [IgM] and elevated serum viscosity in the presence of a lymphoplasmacytic infiltrate in thebonemarrow.Thetreatmentischemotherapy(ChlorambucilorFludarabine).

Dr.KhalidYusufElzohrySohagTeachingHospital2012 67

www.MRCPass.com

Rheumatology

RheumatologyQ063
A 72 year old woman complained of pain at the base of her right thumb. There was tenderness and swelling of the right first carpometacarpal joint. Finkelsteins test is negative. Whatisthemostlikelydiagnosis?
A.Avascularnecrosisofthescaphoid B.DeQuervain'stenosynovitis C.Osteoarthritis D.Psoriaticarthritis E.Rheumatoidarthritis

Answer:C)osteoarthritis. The tenderness at the carpometacarpal joint is most likely due to osteoarthritis in a patientofthisage.

Dr.KhalidYusufElzohrySohagTeachingHospital2012 68

www w.MRCPass.c com

Rheum matology

Rheumatol logyQ064 4
A 45 4 year old man presents to the e opthalmo ologist and was found d to have anterior a uveitis.Uponenquiry, e hehasmouthulcersandarashonhisleg. Wha atisthelike elydiagnosi is?
A.ALam myloidosis B.Familialmediterra aneanfever C.Behce et'sdisease D.Polym myalgiarheumatica E.SLE

Answer:c)Beh hcet'sdiseas se. Beh hets disease may cause ant terior or posterior uveitis, co orneal ulceration, conj junctivitis, papillitis, SVC and IVC I thromb bosis, pulm monary em mbolism, er rythema nod dosum,pust tularlesions s,acneiform mnodules,pathergy p an ndoralulceration.

AnteriorUveitisinBehcet'sdisease

Dr.Khalid K Yusuf fElzohrySo ohagTeachin ngHospital 2012 69

www.MRCPass.com

Rheumatology

RheumatologyQ065
In a patient with SLE, the risk of cardiovascular abnormality is increased with presence ofwhichoneofthefollowing?
A.Anticardiolipinantibody B.AntiRoantibody C.AntiLaantibody D.ANAantibody E.AntiJoantibody

Answer:B)antiRoantibody AntiRoisimportantinpregnancysinceitisassociatedwithbabiesbornwithcongenital heartblock.

Dr.KhalidYusufElzohrySohagTeachingHospital2012 70

www.MRCPass.com

Rheumatology

RheumatologyQ066
A 50 year old secretary noticed tingling and numbness over the palmar surface of her thumb, index and middle fingers after several hours at her computer. Pain in the same areasoftenoccuratnight. Whatisthediagnosis?
A.Rheumatoidarthritis B.Gout C.Dermatomyositis D.Carpaltunnelsyndrome E.Psoriaticarthritis

Answer:d)Carpaltunnelsyndrome. The diagnosis is carpal tunnel syndrome, and the symptoms are due to tenosynovitis whichisworsenedbyrepetitivestrainimposedbytyping.

Dr.KhalidYusufElzohrySohagTeachingHospital2012 71

www.MRCPass.com

Rheumatology

RheumatologyQ067
A 75 year old man complains of pain and stiffness in both his shoulders. He has lost 1/2 a stone in last 6 weeks due to loss of appetite. Investigations show : ESR 95 mm/hr, normochromicnormocyticanaemiaandapositiverheumatoidfactor. Thelikelydiagnosisis:
A.Fibromyalgia B.Polymyositis C.Dermatomyositis D.PolymyalgiaRheumatica E.Chronicmyeloidleukaemia

Answer:d)PolymyalgiaRheumatica. PolymyalgiaRheumaticaisassociatedwithweightloss,anemia&malaise. It can also be associated with a false positive rheumatoid factor, especially in the elderly.

Dr.KhalidYusufElzohrySohagTeachingHospital2012 72

www.MRCPass.com

Rheumatology

RheumatologyQ068
A 12 year old boy has fevers. He has had elbow , shoulder and hip and knee pains over the last few months. On examination, he has a temperature of 37.5 C and palpable hepatosplenomegaly. InvestigationsshowaCRPof30andnegativeANA. Whatisthelikelydiagnosis?
A.Marfan'ssyndrome B.EhlerDanlossyndrome C.Perthe'sdisease D.Juvenilechronicarthritis E.Osteoarthritis

Answer:d)juvenilechronicarthritis. Juvenile chronic arthritis (JCA) is a form of seronegative arthritis in the young (adult form known as Stills disease) which is rheumatoid factor and ANA negative. Diagnostic criteria include high fever, hepatomegaly, splenomegaly, lymphadenopathy, serositis (pleuritis,pericarditis),leucocytosis.Bonedestructionandmicrognathiaoccurs.

Dr.KhalidYusufElzohrySohagTeachingHospital2012 73

www.MRCPass.com

Rheumatology

RheumatologyQ069
A25yearoldladypresentstoclinicwithapainfullyswollenhotrightknee.Shegivesa2 week history of migratory polyarthritis and urethritis. Clinical examination reveals a pustular skin rash, right Achilles tendinitis and left plantar fasciitis. She is HLAB27 negative. Whatisthemostlikelydiagnosis?
A.Relapsingpolychondritis B.Reiterssyndrome C.Gonoccocalarthritis D.Nonspecificurethritis E.Behcet'ssyndrome

Answer:c)Gonoccocalarthritis. Gonoccocal arthritis classically presents with a hot joint on a background of a migrating polyarthropathy. It affects w omen more frequently than men (4:1) and its highest incidence is among sexually active adolescent girls. There is also increased risk during menstruationandpregnancy. Twoformsofarthritisexistonewithskinrashesandmultiplejointinvolvement,anda second, less common, form in which disseminated gonococcemia leads to infection of a singlejoint(monoarticular)andjointfluidculturesarepositive.

Dr.KhalidYusufElzohrySohagTeachingHospital2012 74

www.MRCPass.com

Rheumatology

RheumatologyQ070
A50yearoldwomanwithlongstandingrheumatoidarthritis,andasiccasyndrome.She presentswithincreasingmalaise.Investigationsshowed: Hb7.5g/dl Neutrophilcount1.2x10^9/l plateletcount90x10^9/l HerESRwas120mm/hrandCRP145mg/dl Onexamination,shewaspaleandhadpalpablesplenomegalyof6cmedge. Themostlikelycauseofherhaematologicabnormalitiesis:
A.Anaemiachronicdisease B.UpperGIbleed C.Felty'ssyndrome D.Folatedeficiency E.Irondeficiency

Answer:c)Felty'ssyndrome. Felty'ssyndromeisthetriadofseropositivearthritis,splenomegalyandneutropenia. The cause of Felty's syndrome is not known, but is most commonly associated with rheumatoidarthritis.

Dr.KhalidYusufElzohrySohagTeachingHospital2012 75

www.MRCPass.com

Rheumatology

RheumatologyQ071
32 year old lady upper middle class white lady who used to be an athlete has been referred for investigation due to tiredness. She mentions fatigue which is chronic and occurs even after minor physical w ork. This has been going on for 3 years. Investigations including CK, ESR, EMGs and muscle biopsy have revealed no obvious medicalcauseforthis. Whichofthefollowingisthebesttreatment?
A.Erythropoietininjections B.Cognitivebehaviouraltherapy C.Gradedexerciseprogramme D.Codeine E.Fluoxetine

Answer:C)Gradedexerciseprogramme. Chronic Fatigue syndrome is defined by symptoms and not signs. The clinical profile of an individual with CFS is of a highachieving student or athlete who usually is female (80%), white, and middleclass to upper middleclass. Treatment is largely supportive and responsive to symptomatology. This includes physical therapy and modest aerobic or anaerobic exercise (if possible) to avoid cardiovascular deconditioning. Sleep may be addressed with medication; often, melatonin or nighttime amitriptyline is helpful. If presentandsevere,painoftenisaddressedinapainclinic.

Dr.KhalidYusufElzohrySohagTeachingHospital2012 76

www w.MRCPass.c com

Rheum matology

Rheumatol logyQ072 2

A 45 5 year old patient p pres sents with p proximal muscle weakness, partic cularly in th he lower limb bs.Shehasaheliotropicrasharou undtheeye esandalsoGottrons G papules. Whi ichoneofth hefollowing gantibodie esismostst tronglyasso ociated?
A.La B.Ro C.Jo1 D.SCL70 7 E.AntiDsDNA D

Answer:C)Jo1. 1 The e diagnosis is dermato omyositis. Anti Jo1 antibody a is associated with acute onset myo ositis, particularly dermatomyositis. The lim mb girdle or proximal muscles ar re most seve erelyaffectedinbothp polymyositi isandderm matomyositis.

Got ttron'spapu ules

Dr.Khalid K Yusuf fElzohrySo ohagTeachin ngHospital 2012 77

www.MRCPass.com

Rheumatology

RheumatologyQ073
A 9yearold boy is brought to the paediatricianwith complaints of fever for 1 week. He alsohascrackedlips,whicharepainful. On examination, his eyes are red and he has a lymph node in the cervical region. Kaw asakisyndromeissuspected. Whichofthefollowingisthebesttreatmentchoice?
A.Intravenousimmunoglobulim B.Hydrocortisone C.Abxicimab D.Amoxicillin E.Lowmolecularweightheparin

Answer:a)Intravenousimmunoglobulin. This is a case of Kaw asakis syndrome, which is also known as mucocutaneous lymph node syndrome and occurs mainly in children under 10 years of age. It is a form of vasculitis, which affects coronary arteries and is associated with the development of coronaryaneurysms. Aspirin and IV immunoglobulin at high doses is the treatment of choice. Corticosteroids used to be contraindicated, but recently trials have shown that it reduced the risk of heartdamagecausedbycoronaryvesselvasculitis. Infliximab is also beingused in trials. Warfarin is used if coronary aneurysms develop as acomplication.

Dr.KhalidYusufElzohrySohagTeachingHospital2012 78

www.MRCPass.com

Rheumatology

RheumatologyQ074
A55yearoldwomanpresentswitha2monthhistoryofpainaffectingthecervicalspine and both shoulders, this was preceded by pain in the lower back and both hips. Early morning stiffness lasts until lunchtime and she feels markedly tired and weak. Examinationrevealsbilateralkneeeffusionsandarightcarpaltunnelsyndrome. Investigations demonstrate a normochromic nomocytic anaemia of Hb 9.8 g/dl, ESR 72 mm in the first hour, CRP 12 g/l, serum immunoglobulins and protein electrophoresis show a polyclonal increase in gamma globulins and elevated alpha 1 and alpha 2 globulinsbutnoparaproteinband. Whatisthemostlikelydiagnosis?
A.Rheumatoidarthritis B.Paraneoplasticsyndrome C.Mixedconnectivetissuedisease D.Scleroderma E.Polymyalgiarheumatica

Answer:e)Polymyalgiarheumatica. Polymyalgia rheumatica (PMR) is a clinical diagnosis based on pain and stiffness of pelvicgirdleandshoulder.Itismorecommonaftertheageof55. Incontrasttopolymyositisthereisnomuscularweakness.Earlymorningstiffnessofthe hipandshouldergirdlesiscommon.Anormochromicnormocyticanaemiaisassociated.

Dr.KhalidYusufElzohrySohagTeachingHospital2012 79

www.MRCPass.com

Rheumatology

RheumatologyQ075
A55yearoldmanpresentswithsuddenonsetofarightsidedfootdropwithnumbness over the dorsum of his right foot. He has weakness of dorsiflexion and eversion of his rightfoot.Thereisanareaofdiminishedsensationoverthedorsumoftherightfoot.He also has a left wrist drop with loss of sensation over the dorsal aspect of the first interosseus space. For the last tw o months, he has lost weight and he had generalized myalgia. Investigationsreveal: Hb11.5g/dL WBC13x10^9/l Neutrophils9.2x10^9/l Lymphocytes2.2x10^9/l ESR60mm/1sthour Urinalysis:Protein++,Blood++ Themostlikelydiagnosisis:
A.Tuberculosis B.Polyarteritisnodosa C.Sarcoidosis D.Rheumatoidarthritis E.Systemiclupuserythematosus

Answer:b)Polyarteritisnodosa. The patient has a systemic illness with mononeuritis multiplex and renal involvement. Although all the listed conditions can cause mononeuritis, PAN is one of the few connective tissue disorders that usually occur in middleaged men unlike RA and SLE, whicharecommonerinfemales.

Dr.KhalidYusufElzohrySohagTeachingHospital2012 80

www w.MRCPass.c com

Rheum matology

Rheumatol logyQ076 6
A 60 0 year old man m presen nts with pain in his left t foot. He mentions m tha at he has had poor mob bility.Generalexamina ationrevealsaswollen nankle.Neu urologicalex xaminationreveals abse ent ankle je erk and wea ak foot flexion/extensi ion on the left. l His ESR R is 30 mm/ /hr, CRP is<5 5mg/l. Joint t xray reve ealssubchon ndralfractu ures,softtis ssueswellin ng and a narrowed jointspace. nosisis: Thelikelydiagn
A.Charc cot'sjoint B.Osteo oarthritis C.Gout nilechronicarthritis a D.Juven E.Previo ousroadtraf fficaccident

Answer:a)Cha arcot'sjoint. Cha arcot joint or neurop pathic joint disease ca an be caus sed by dia abetic neur ropathy, syph hillis, syring gomyelia or o leprosy. The X ray changes are a reduced d joint space with peri iarticular destruction, d , subchond dral bodies s, loose bo odies and occasionally joint dislo ocation.

Cha arcot'sjoint

Dr.Khalid K Yusuf fElzohrySo ohagTeachin ngHospital 2012 81

www.MRCPass.com

Rheumatology

RheumatologyQ077
A55yearoldmanpresentswithlethargy,polyuria,polydipsiaandstiffnessofthehands. Hehasevidenceofanarthopathyaffectingthe2nd&3rdmetacarpophalangealjointsof bothhands.Xrayconfirmsevidenceofdegenerativediseaseatthesesites. Whichofthefollowingthelikelydiagnosis?
A.Behcet'ssyndrome B.Haemochromatosis C.Amyloidosis D.Reiter'ssyndrome E.Osteoarthritis

Answer:b)haemochromatosis. The characteristic feature of haemochromatotic arthropathy is involvement of the secondandthirdmetacarpals.Thearthropathycanleadtoextensivejointdestruction. These patients may have episodes of acute, inflammatory pseudogout from such deposition.

Dr.KhalidYusufElzohrySohagTeachingHospital2012 82

www.MRCPass.com

Rheumatology

RheumatologyQ078
An 22 year old girl presents with a 6 week history of polyarthralgia with early morning stiffness. Her symptoms responded well initially to Ibuprofen but she then they worsen again. She is otherw ise well apart from a history of acne which well controlled on Minocycline.Hermothersevererheumatoidarthritis. Investigations show : ESR 40 mm/hr, CRP 110 mg/l, rheumatoid factor negative, ANA stronglypositive(1:1600),antidsDNAantibodiesnegative. Whatisthelikelycauseofhersymptoms?
A.Druginducedlupus B.SystemicLupusErythematosus C.Psoriaticarthropathy D.Juvenilechronicarthritis E.Fibromyalgia

Answer:a)Druginducedlupus. The likely diagnosis is drug induced SLE. Minocycline is one of the causes well documentedasacauseofdruginducedSLE. Characteristically,inflammatorymarkerssuchasESR&CRParebothmarkedlyelevated. ANA can be strongly positive but AntidsDNA antibodies are usually negative. Symptoms usuallyresolvefollowingwithdrawalofthedrugafterseveralmonths.

Dr.KhalidYusufElzohrySohagTeachingHospital2012 83

www.MRCPass.com

Rheumatology

RheumatologyQ079
A 50 year old woman presents to casualty with a 2 day history of pain and swelling of theleftankle.Shedeniesanyhistoryofrecenttrauma.Onexamination,shewasfebrile, temperature 38.3 C. The left ankle was swollen and very tender with a reduced range ofmovement. Whichofthefollowinginvestigationswouldbemosthelpful?
A.Uratelevel B.ESR C.Rheumatoidfactor D.Aspirationofsynovialfluid E.KneeXray

Answer:d)aspirationofsynovialfluid. Joint fluid examination is important in excluding septic arthritis, and can also help to confimgoutorpseudogout.

Dr.KhalidYusufElzohrySohagTeachingHospital2012 84

www.MRCPass.com

Rheumatology

RheumatologyQ080
A 35 year old woman with newly diagnosed rheumatoid arthritis enquires about risk factors. Whichoneofthefollowingisariskfactorforrheumatoidarthritis?
A.Pneumococcalinfection B.HLADR4 C.SLE D.HIV E.Renalfailure

Answer:b)HLADR4. Riskfactorsforrheumatoidarthritisare: HLADR4 EBV parvovirusB19andrubellainfections bloodtransfusion smoking(inducesRFproduction) stress obesity PregnancyandOCPareprotective.

Dr.KhalidYusufElzohrySohagTeachingHospital2012 85

www.MRCPass.com

Rheumatology

RheumatologyQ081
A26yearoldmanpresentswitha6monthhistoryoflowbackpain.Thepainradiatesto his buttocks. There is associated stiffness which is w orse in the morning and after periodsofinactivity. Whichofthefollowingsignsismostlikelytobepresent?
A.Footdrop B.Positivefemoralstretchtest C.PositiveTrendelenburgtest D.Sacroiliacjointtenderness E.Kernigssign

Answer:D)sacroiliacjointtenderness. The diagnosis is ankylosing spondylitis as this is a young patient with lower back pains andmorningstiffness.Sacroilitisisacommonmanifestation.

Dr.KhalidYusufElzohrySohagTeachingHospital2012 86

www w.MRCPass.c com

Rheum matology

Rheumatol logyQ082 2
A62 2yearoldman m hascom mplainsofapaininthe etemples,fevers, f swea atsandmalaisefor 4 weeks. w The right r side of f the face w was mildly swollen s and d he experie enced visua al loss in therighteye. Onexaminatio e n,hehadp prominenta andtendertemporal t ar rteriesonth heright. Bloo odtestsrev vealed: Hb1 12.5g/dl MCV86fl WCC12x10^9 9/l telets212x10^9/l plat /hr ESRof90mm/ atisthelike elydiagnosi is? Wha
A.Pituit tarytumour B.Grave e'sdisease C.Paget t'sdisease D.Catar ract E.Temp poralarteritis s

Answer:e)tem mporalarter ritis. The ediagnosisistemporalarteritis(gi iantcellarte eritis). The e complications of Gia ant Cell Art thritis are related to arterial inf flammation n. These include: blindness absentpuls ses hypertensio on angina stroke claudication

Prominenttem mporalarter ries


Dr.Khalid K Yusuf fElzohrySo ohagTeachin ngHospital 2012 87

www w.MRCPass.c com

Rheum matology

Rheumatol logyQ083 3
A 60 0 year old woman w pre esents with a history of o progressive difficulty y in climbin ng stairs andrisingfrom mchairs. ficulty in sw w allow ing. She compl lains that her hands discolour She also complains of diff andbecomeco oldeasily. On examinatio on she has purple disc colouration of her eye elids and periorbital oedema. o The ereisweakn nessofthemuscles m ofher h limbgir rdles. atisthelike elycauseof fthedyspha agia? Wha
A.Oesophagealcanc cer B.Derm matomyositis C.SystemicLupusEr rythematosu us D.Retro osternalgoitr re E.Sclero oderma

Answer:b)Der rmatomyositis. The e diagnosis is dermato omyositis. Dysphagia in dermato omyositis is s primarily due to wea akness of the striated d musculatu ure of the posterior pharynx. p Dy ysphagia may m also resu ult from cricopharyngeal obstruc ction secondary to inf flammation or fibrosis s of the cricopharyngeu usmuscles.

Heli iotroperash hindermat tomyositis

Dr.Khalid K Yusuf fElzohrySo ohagTeachin ngHospital 2012 88

www.MRCPass.com

Rheumatology

RheumatologyQ084
A 60 year old man is on frusemide for leg oedema. He presents with a painful swollen knee. His temperature is 38 C and he has a white cell count of 12 x 10^9/l and CRP of 120mg/l.Uricacidis0.49mmol/l(<0.42). Ajointaspiraterevealspuscellsandnegativelybirefringentcrystalswithpolarizedlight. AftertreatmentwithNSAIDsfor48hourshehasnotimprovedandtheswellingpersists. Therearenoorganismsculturedfromthejointaspirateorbloodcultures. Whatisthebestcourseofaction?
A.Allopurinol B.MRItolookforosteomyelitis C.Intravenousflucloxacillin D.Arthroscopyandwashout E.Furtherjointaspirationwithdepomedroneinjection

Answer:e)furtherjointaspirationwithdepomedroneinjection. This is a case of acute gout which has not settled on medical therapy (NSAIDs or colchicine). The raised inflammatory markers can be due to acute gout. Uric acid levels are not necessarily high in acute gout. A depomedrone (steroid) injection with further aspirationofjointfluidisthenextstep.

Dr.KhalidYusufElzohrySohagTeachingHospital2012 89

www.MRCPass.com

Rheumatology

RheumatologyQ085
A 40 year old lady presents with polyuria and thirst. Her serum calcium of 2.85 mmol/l andaparathyroidhormoneof12(18)pmol/l.Shehasnormalrenalfunction. Whichtestprovidesthebestassessment?
A.CTofherspine B.DualenergyXrayabsorptiometry C.VitaminDlevels D.Isotopebonescan E.Urinebencejonesprotein

Answer:b)DualenergyXrayabsorptiometry. This patient is likely to have primary hyperparathyroidism (secondary to a parathyroid adenoma).Thebestassessmentistodeterminetheseverityoflossofbonedensitywith aDEXAscaninordertoconsiderparathyroidsurgeryisnecessary.

Dr.KhalidYusufElzohrySohagTeachingHospital2012 90

www w.MRCPass.c com

Rheum matology

Rheumatol logyQ086 6
An 4 45 year old man has had worseni ing back pains, shoulde er pains and right hip pain for 8 ye ears. The pa ain is typica ally w orse a at the end of day. He also has bo ony enlargem ment of the distal inte erphalangea al joints. An X ray of f the shoul lder reveals the prese ence of prominent oste eophytes. There T is sclerosis and narrow ing g of the join nt space at the hip jointsonthepe elvicXray. atisthelike elydiagnosi is? Wha
A.Osteo omyelitis B.Osteo oarthritis C.Osteo omalacia D.Rheumatoidarthritis E.Paget t'sdisease

Answer:b)oste eoarthritis. The e clinical his story sugges sts early on nset osteoar rthritis (which can be idiopathic or o occur in athletes). a Th he X ray fea atures of osteophytes, sclerosis and a narrow w ing of join nt space aresuggestiveofosteoart thritis.

Oste eophyteon nashoulder rXray

Dr.Khalid K Yusuf fElzohrySo ohagTeachin ngHospital 2012 91

www.MRCPass.com

Rheumatology

RheumatologyQ087
A 35 year old woman has generalized joint pains and muscle aches. She also complains of gritty sensations in the eye in the mornings. Blood tests show Anti Nuclear Antibody ++,RheumatoidFactor++. Whatisthediagnosis?
A.Polylmyositis B.Reiterssyndrome C.PrimarySjogrenssyndrome D.Polyarteritisnodosa E.Rheumatoidarthritis

Answer:C)PrimarySjogrenssyndrome. Dry mouth, dry eyes, fatigue, muscle aches and joint pains are typical of Sjogrens syndrome. 80% of patients are ANA positive and 75% are Rheumatoid factor positive. A usefuldiagnostictestistheSchirmer'stest,whereapieceoffilterpaperisplacedinthe corneroftheeyetomeasurethedegreeofwettingafterfiveminutes.

Dr.KhalidYusufElzohrySohagTeachingHospital2012 92

www.MRCPass.com

Rheumatology

RheumatologyQ088
A 60 year old woman presents with a tw o week history of malaise and lower limb joint pains. On examination she had a vasculitic rash over her shins, thighs and buttocks. Investigationsrevealed: Hb10.2g/dL platelets265X10^9/L creatinine380mol/L antinuclearantibodiesNegative antineutrophilcytoplasmicantibodiesNegative antiglomerularbasementmembraneantibodiesNegative dipstixurinalysisblood+++ protein+ Whatisthelikelydiagnosiscausingrenalimpairment?
A.Psoriaticarthritis B.HenochSchonleinpurpura C.Polymyositis D.Membranousnephropathy E.Myeloma

Answer:b)HenochSchonleinpurpura. The distribution of the rash together with lower limb joint pains are suggestive of HenochSchonleinpurpura.Thisusuallyoccursinchildrenaged210yearsbutcanoccur in older age groups. The only w ay of differentiating this condition from other small vesselvasculitidesisbybiopsy.ThiswouldshowIgAdepositioninvesselwallsondirect immunofluorescence.

Dr.KhalidYusufElzohrySohagTeachingHospital2012 93

www.MRCPass.com

Rheumatology

RheumatologyQ089
A 28 year old woman presents with a right knee joint pain and a 4 month history of weight loss. She thinks she has lost weight because of diarrhoea, which occurs several timesaday.Examinationrevealsaswollen,tenderrightkneejointwithasmalleffusion. Thelikelydiagnosisis:
A.Reiter'ssyndrome B.Inflammatoryboweldisease C.Behcet'sdisease D.Campylobacterinfection E.Rheumatoidarthritis

Answer:b)Inflammatoryboweldisease. The description of weight loss, diarrhoea and a mono/oligoarthropathy suggests a diagnosis of inflammatory bow el disease. Peripheral arthritis, peripheral arthralgia withoutjointswellingoreffusion,degenerativejointdiseaseorseropositivearthritiscan occur in inflammatory bow el disease. In patients with peripheral arthralgia and peripheral arthritis, there is a significantly greater prevalence of mucocutaneous manifestations of IBD i.e. oral ulceration, erythema nodosum, pyoderma gangrenosum, anduveitis.

Dr.KhalidYusufElzohrySohagTeachingHospital2012 94

www.MRCPass.com

Rheumatology

RheumatologyQ090
A professional tennis player presents with shoulder pains especially whilst serving the ball. He has limited passive and active shoulder abduction to less than 60. His temperatureis36.5Candhehasanormalwhitecellcount.Thereistendernessaround theanteriorportionoftheshoulderjoint. Whichdiagnosisislikely?
A.Glenohumeraljointosteoarthritis B.Bursitis C.Tenniselbow D.Supraspinatustendonitis E.Septicarthritis

Answer:d)supraspinatustendonitis. Pain during abduction with limitation of movement is suggestive of supraspinatous tendonitis. Palpation or compression around the greater tubercle of the humerus is particularlytender.

Dr.KhalidYusufElzohrySohagTeachingHospital2012 95

www.MRCPass.com

Rheumatology

RheumatologyQ091
A 45 year woman has raynauds phenomenon. She also has difficulty insw allow ing and dyspnoea. Echocardiographyshowsrightheartstrain.Bloodtestsrevealrenalimpairment. Whichoneofthefollowingantibodiesisspecifictothislady'scondition?
A.Anticentromereantibody B.TopoisomeraseI C.AntidsDNAantibody D.AntiSCL70antibody E.Antimitochondrialantibody

Answer:d)antiSCL70antibody. AntiSCL70antibody(topoisomeraseI)istypicallyfoundinprogressivesystemicsclerosis (notthelimitedcutaneousform,CREST).

Dr.KhalidYusufElzohrySohagTeachingHospital2012 96

www.MRCPass.com

Rheumatology

RheumatologyQ092
A 55 year old man presents with a 6 week history of lethargy and diffuse purpuric rash. He is noted to have a right foot drop and a left ulnar nerve palsy. He complains of arthralgia but has no clinical evidence of inflammatory joint disease. Echocardiogram is unremarkable, blood cultures are negative, ESR 80 mm/hr, ANCA negative, ANA negative, rheumatoid factor strongly positive, C3 1.1 g/l (0.75 1.6), C4 0.03 g/l (0.14 0.5). Dipstickurinalysisshowsblood++. Whatisthelikelydiagnosis?
A.Takayasu'sarteritis B.ANAnegativeSLE C.Culturenegativeendocarditis D.Cryoglobulinaemia E.Rheumaticfever

Answer:d)Cryoglobulinaemia. A low C4 together with a strongly positive rheumatoid factor suggests cryoglobulinaemia as a cause of mononeuritis multiplex and rash. Palpable purpura, arthralgia, hepatosplenomegaly, diffuse proliferative glomerulonephritis, Raynaud's phenomenonandthrombosismayoccur. Type I cryoglobulinaemia may be associated with lymphoproliferative disorders, multiple myeloma, and monoclonal gammopathy of uncertain significance, and macroglobulinaemia.Plasmapheresismayreducethelevelsofcryoglobulin. Type II cryoglobulinaemia (mixed monoclonal) is usually composed of a monoclonal component (usually IgG, IgM or IgA) and a polyclonal component (mainly IgG). Causes are connective tissue diseases, Hepatitis B and C infection, infectious mononucleosis andlymphoma.

Dr.KhalidYusufElzohrySohagTeachingHospital2012 97

www.MRCPass.com

Rheumatology

RheumatologyQ093
A 70 year old woman with a history of multiple myeloma has a serum calcium of 2.9 mmol/l.Heisprescribedpamidronateinfusionover4days. Whatisitsmechanismofaction?
A.Promotescalcitonin B.Increasescalcitriollevels C.Inhibitosteoclasts D.inhibitosteoblasts E.Stimulateparathyroidhormonesecretion

Answer:C)inhibitosteoclasts. Bisphosphonates inhibits osteoclasts and reduces progression tow ards bone destruction.

Dr.KhalidYusufElzohrySohagTeachingHospital2012 98

www.MRCPass.com

Rheumatology

RheumatologyQ094
A 80 year old lady presents with a 5 day history of severe left temporal headache radiating from her eye to the scalp. She had also experienced jaw discomfort during eating. Whichofthefollowingdrugsshouldbegivenwhileawaitingresultsofdiagnostictests?
A.Carbamazepine B.Prednisolone C.Azathioprine D.Infliximab E.Intravenousimmunoglobulin

Answer:b)Prednisolone. The history suggests temporal arteritis. In view of the vision threatening nature of disease, the patient should be commenced on steroids. Typically 60mg of prednisolone perdayisrecommended.

Dr.KhalidYusufElzohrySohagTeachingHospital2012 99

www w.MRCPass.c com

Rheum matology

Rheumatol logyQ095 5
A 30 0 year old woman w has s a 3 month h history of arthralgia. There is sw welling of th he distal inte erphalangea al joints of t the ring fingers of the hand. The wrist on th he right and d ankles are swollen as s well. Ony ycholysis w was noted on the nails. Her serum inflam mmatory mar rkersareraised. ichofthefo ollowingisthe t likelydia agnosis? Whi
A.Polym myalgiarheumatica B.Rheumatoidarthr ritis C.Gout aticarthropa athy D.Psoria E.SLE

Answer:d)Pso oriaticarthro opathy. Psoriaticarthritiseffectsd distalinterp phalangealjoints j tends sbeasymm metrical. The ereare5typ pesofpsoria aticarthritis. Asymmetrical oligoarticular arthritis is thought t to be the most comm mon type. Usually, the digits of the t hands and feet a are affected d first, with h inflamma ation of the e flexor tend don and synovium s occurring o s simultaneou usly, leadin ng to the typical "sa ausage" app pearance(da actylitis). The e other type es are symmetrical po olyarthritis, DIP arthro opathy, arth hritis mutila ans and spondylitiswithorwithou utsacroilitis s.

Psoriaticarthro opathy
Dr.Khalid K Yusuf fElzohrySo ohagTeachin ngHospital 2012 100

www.MRCPass.com

Rheumatology

RheumatologyQ096
A 60 year old lady has severe rheumatoid arthritis. She is currently on Methotrexate 20 mg weekly for the past 5 months and also has been receiving regular infusions of Infliximab. Her joint disease dramatically improved. She now presents with fevers, coughandthereisevidencealargeleftsidedpleuraleffusiononherCXR. Whatisthelikelydiagnosis?
A.Methotrexatepneumonitis B.CMVinfection C.Tuberculosis D.Bronchialcarcinoma E.Rheumatoidrelatedpulmonaryfibrosis

Answer:c)tuberculosis. Serious opportunistic infections have been associated with the anti TNF alpha drug infliximab, but the frequency of TB exceeds that associated with other infections. Infliximabmayincreasetheriskoflymphoma.

Dr.KhalidYusufElzohrySohagTeachingHospital2012 101

www.MRCPass.com

Rheumatology

RheumatologyQ097
A 75 year old man presents with an acute onset of severe pain and swelling of the left elbow.Hementionsthathehadachestinfection3weeksago.Onexamination,hehad atemperatureof38Candtheleftelbowwaserythematous,swollenandtender. Whatisthemostappropriateinvestigation?
A.Creactiveprotein B.Fullbloodcount C.Jointaspiration D.Uricacidlevel E.Xrayofthejoint

Answer:C)jointaspiration. This patient is likely to have reactive arthritis. How ever, gram stain and culture are necessarytoexcludesepticarthritis.

Dr.KhalidYusufElzohrySohagTeachingHospital2012 102

www.MRCPass.com

Rheumatology

RheumatologyQ098
A 75 year old man has significant bony pains which have been occuring for 3 years. He presents to the clinic for assessment and the investigations results were obtained below: Correctedcalcium2.4(2.22.6) ESR20mm/1sthr Alkalinephosphatase625iu/L(50100) gammaGT42iu/L(1050) Prostatespecificantigen7.4pg/L(06) Whatisthemostlikelydiagnosis?
A.Osteoporosis B.Osteomalacia C.Metastaticprostaticcarcinoma D.Paget'sdisease E.Multiplemyeloma

Answer:d)Paget'sdisease. Paget'sdiseasecausesahighalkalinephosphataseandnormalcalciumlevels. Paget's Disease represents an imbalance of bone formation and resorption. It typically begins with excessive bone resorption followed by excessive bone formation. The main disturbanceisanexaggerationofosteoclasticboneresorption. The most common sites of involvement include the spine, pelvis, skull, femur and tibia. Skull involvement may produce enlargement ofthe headcharacterizedby more evident frontalbossinganddilatedsuperficialcranialmuscles. Conductive and/or sensorineural hearing loss may result from disease of the temporal boneorossicles.

Dr.KhalidYusufElzohrySohagTeachingHospital2012 103

www.MRCPass.com

Rheumatology

RheumatologyQ099
A 75 year old patient with osteoarthritis of the knee has been taking codeine 30mg qds andalsoparacetamolregularly.Hecontinuestohavekneepains. Whatisthenextbesttreatment?
A.Diclofenac B.Morphine C.Ibuprofengel D.Higherdoseofcodeine E.Oralhydrocortisone

Answer:C)ibuprofengel. NSAIDS tend to be better for pain control in osteoarthritis, but have significant side effects. Local NSAID application should be considered as well as intraarticular steroid injections.

Dr.KhalidYusufElzohrySohagTeachingHospital2012 104

www w.MRCPass.c com

Rheum matology

Rheumatol logyQ100 0
A 40 year old lady presents with a swollen rig ght knee. This was asp pirated. Un nder the pola arised microscope, the ere were neutrophils +++ and so ome blue rh homboidal crystals und derparallelpolarisedlig ght. ydiagnosisis: i Themostlikely
A.Osteo oarthritis B.Rheumatoidarthr ritis C.Gout D.Pseud dogout E.Septic carthritis

Answer:d)Pse eudogout. The e blue colo our and po ositively birefringent crystals un nder polarised light suggest pseudogout.Neutrophille evelsarehig ghininfecti ion,goutan ndpseudogout.

Dr.Khalid K Yusuf fElzohrySo ohagTeachin ngHospital 2012 105

www.MRCPass.com

Rheumatology

RheumatologyQ101

A 45 year old man has a renal transplant. He presents with a hot swollen ankle on the leftandfevers.Thesymptomshavebeenpresentfor2days. Whatshouldbedone?
A.Checkserumuricacidlevel B.Injectintraarticularsteroids C.Givecolchicine D.Sendjointfluidformicroscopyandculture E.Intravenouscefuroxime

Answer:d)Sendjointfluidformicroscopyandculture. The differential diagnosis is gout (common in renal patients) but the patient will be in immunosupressionandsepticarthritisshouldbeexcludedwithjointfluidculture.

Dr.KhalidYusufElzohrySohagTeachingHospital2012 106

www w.MRCPass.c com

Rheum matology

Rheumatol logyQ102 2
A40 0yearoldathlete a hasp painonabd ductionofher h arm,par rticularlywhenresiste ed. ichtendonpathology p is i affected? Whi
A.Bicep pstendonitis B.Supra aspinatusten ndonitis C.Teres sminortendo onitis D.Infras spinatustend dinitis E.Subsc capularistendonitis

Answer:b)Sup praspinatustendonitis. Pain ninabductionupto90 0degreesis sduetosup praspinatustendonitis.

Illus strationofsupraspinat s usmuscle


Dr.Khalid K Yusuf fElzohrySo ohagTeachin ngHospital 2012 107

www.MRCPass.com

Rheumatology

RheumatologyQ103
A 60 year old man has worsening discomfort in both shoulders. He is haemodialysis dependent. Past medical history included bilateral carpal tunnel decompression. His Investigations reveal: haemoglobin 9.8 g/dl, ESR 35 mm/1st hr, C reactive protein 15 mg/L,Urate0.58. Whatisthelikelydiagnosis?
A.Reiter'ssyndrome B.Amyloidosis C.Polymyalgiarheumatica D.Gout E.Osteomalacia

Answer:b)amyloidosis. b2 microglobulin deposition in joints may lead to amyloidosis. This can occur 10 years onfromdialysis,andleadtocarpaltunnelsyndrome.

Dr.KhalidYusufElzohrySohagTeachingHospital2012 108

www.MRCPass.com

Rheumatology

RheumatologyQ104
A 45 year old man presents with a painful swollen knee. He feels generally unwell and has fever. He has recently had a flulike illness, an erythematous rash on the trunk followed by a selflimiting episodeof diarrhoea. A diagnosis of reactive arthritis is made bytherheumatologist. Howshouldthepatientbemanaged?
A.Highdosesteroids B.Broadspectrumivantibiotics C.Arthroscopyandwashoutofthejoint D.Ifthesymptomsbecomeschronic,sulphasalazinemaybeuseful E.Bonescantolookforafocusofinfection

Answer:d)ifthesymptomsbecomeschronic,sulphasalazinemaybeuseful. The diagnosis is likely to be reactive arthritis following an infectious illness. Although joint aspiration may be useful, there is no need for arthroscopy at present. NSAIDs may be used for symptoms control now. If the symptoms persist, sulphasalazine or methotrexatemaybeuseful.

Dr.KhalidYusufElzohrySohagTeachingHospital2012 109

www w.MRCPass.c com

Rheum matology

Rheumatol logyQ105 5
A16 6girlisinve estigatedfo orswellinga andpainintherightwrist, w leftkneeandrigh htankle. Inve estigationsshow s apositiveANA1:160withve Rheuma atoidfactor. . Wha atissheatriskofdeve eloping?
A.Psoria asis B.Butte erflyfacialras sh C.Erosiv vejointdisea ase D.Uveit tis E.Bamb boospine

Answer:D)uve eitis. Juve enilechroni icarthritisisatermusedtodescr ribearthritis soccurringinsomeone ewhois less s than 16 years old that lasts for r more than n three mo onths. Large e joints tend to be affe ected.Rheumatoidfact torisoften negative,and a thereispositivean ntinuclearantibody especially e in pauciart ticular JCA. . Acute anterior uve eitis is most commonly in pauciarticular juvenile ch hronic arthritis. Stiffne ess, amyloi idosis, and osteoporo osis also occu ur.

Uve eitiscausing gredeye(le enticularpre ecipitatesmay m beseen n)

Dr.Khalid K Yusuf fElzohrySo ohagTeachin ngHospital 2012 110

www w.MRCPass.c com

Rheum matology

Rheumatol logyQ106 6
A22 2yearoldman m presentswitha4w weekhistor ryofapainf fulswollen leftknee.He H hasa past t medical history of a treated sexually transmitted disease 6 months ago. a On examinationth herewasalargeeffusio oninthelef ftknee. Synovialfluidanalysis a sho owsawhite cellcountof o 15x10^9 9/lbutcultu urewasneg gative. Whi ichoneofth hefollowing gorganismsisthemos stlikelycau use?
A.Gardn nerella B.Chlam mydia C.Trepo onemapallidum D.Neiss seriagonorrh hoea E.Tricho omonasvaginalis

Answer:d)Neisseriagono orrhoea. Gon nococcal ar rthritis is ca aused by a an infection n with the gramnegat tive diploco occus N gon norrhoeae. Neisseria gonorrhoea g a occurs in n young adults, ofte en preceded by a mig gratoryarthritis. The e bacteremi ic phase is a classic t triad of migratory polyarthritis, tenosynovitis, and derm matitis. This patient was w treated previously y for a sexu ually acquire ed infection n hence may ybetherea asonforthe eculturetobenegative e. The e initial treatment of choice for r gonococc cal arthritis s is a third dgeneration beta lactamaseresi istant ceph halosporin (eg, ceftri iaxone, cef ftizoxime, cefotaxime e) or a pen nicillin,ifthe eorganismissensitive.

Mul ltipleintrac cellulargram mnegativediplococcic d

Dr.Khalid K Yusuf fElzohrySo ohagTeachin ngHospital 2012 111

www.MRCPass.com

Rheumatology

RheumatologyQ107
A 35 year old lady has positive ANA, and has a butterfly shaped rash on her face. Her physician makes a diagnosis of SLE. She has flare ups of joint swellings and pains requiring several months treatment with prednisolone. 1 year later she presents with hippainlimitinghermobility. Whichoneofthefollowingisalikelycause?
A.Rheumatoidarthritis B.Septicarthritis C.Juvenilechronicarthritis D.Avascularnecrosis E.Perthe'sdisease

Answer:d)avascularnecrosis. 15% of patients with SLE develop avascular necrosis of the bone. Nephritis, vasculitis andlongtermsteroidusepredisposetoavascularnecrosis.

Dr.KhalidYusufElzohrySohagTeachingHospital2012 112

www.MRCPass.com

Rheumatology

RheumatologyQ108
A50yearoldmanpresentswithchronicrecurrentsinusitisandoccasionalhaemoptysis. HehasmiddlelobepatchyshadowingonhischestXray.Hiscreatinineis145andurine dipstickshows++proteinandblood. Whichtestresultismostlikely?
A.PositivecANCA B.PositiveANA C.PositiveKveimtest D.PositiveASOT E.SputumAFBpositive

Answer:a)positivecANCA. Nasal cavity, pulmonary and renal granulomatous involvement is classical in Wegener's granulomatosis. 70% of patients have a positive cANCA. In addition, w hen ANCA is positive, PR3 (Wegener's) and MPO (microscopic polyangitis) helps to distinguish Betweenthetwodifferentialdiagnoses.

Dr.KhalidYusufElzohrySohagTeachingHospital2012 113

www.MRCPass.com

Rheumatology

RheumatologyQ109
A 55 year old female presents complaining of bone pains and muscular weakness. Her gaitiswaddlinginnature. Xrayshowspseudofracturesofherpubicrami. Investigations show : Serum corrected calcium 2.05 mmol/L, phosphate 0.43 mmol/L, alkalinephosphatase230U/L. Whatisthediagnosis?
A.Pseudohypoparathyroidism B.Secondaryhyperparathyroidism C.Osteopetrosis D.Osteomalacia E.Paget'sdisease

Answer:d)osteomalacia. The clinical radiological and biochemical features in this patient suggest she has osteomalacia. Osteomalacia is characterized by a low serum calcium and phosphate withelevatedserumalkalinephosphatase. Osteomalacia may be caused by deficiency of vitamin D or phosphate deficiency. Malabsorption syndromes, renal failure and liver disease can result in vitamin D deficiency.

Dr.KhalidYusufElzohrySohagTeachingHospital2012 114

www w.MRCPass.c com

Rheum matology

Rheumatol logyQ110 0
A 22 year old Armenian man m presen nts with pain and swelling of the left knee. He also give es a histor ry of recur rrent episo odes of fev ver, pleuritic pains in the che est, and generalised ra ash. These episodes typically la ast for 23 days. Ex xamination reveals enomegaly,swollenkneesandank kles.Urinedipstick d rev veals2+pro oteinuria. sple Wha atisthelike elydiagnosi is?
A.Ankyl losingspond dylitis B.React tivearthritis C.FamilialMediterra aneanfever D.Tuberculosis E.Dermatomyositis

Answer:c)Fam milialMedite erraneanfe ever. Fam milial Mediterranean fever is an inherited d condition n character rized by re ecurrent epis sodes of pa ainful inflam mmation in the abdom men, chest, or joints. These T episo odes are ofte en accompa anied by fev ver and som metimes a rash. The first episode e usually oc ccurs by the age of 20 years, but t in some cases, the initial attack occurs much m later r in life. Typically, episo odes last 12 2 to 72 hour rs and can vary v in seve erity and in the length of time Betw weenattacks. AA amyloidosis commonl ly involves the kidney ys, spleen a and GI tract. Colchicin ne given prophylactically in FMF of ffers some protection against the e developm ment of amy yloidosis inmost m patient ts. Fam milial Mediterranean fever prim marily affe ects popula ations orig ginating fro om the Med diterranean n region, particularly p people of Armenian, , Arabic, Turkish, T and d North African Jew ish ancestry. . Mutations s in the MEFV gene cause c familial Mediterranean feve er.

Rashseeninfa amilialmedi iterraneanfever f

Dr.Khalid K Yusuf fElzohrySo ohagTeachin ngHospital 2012 115

www w.MRCPass.c com

Rheum matology

Rheumatol logyQ111 1
A 35 year old man presents acutely y with ureth hritis, conju unctivitis an nd arthritis. He has bee enhavingjointpainsinthewristandhips. n,hehasarashonthe Onexaminatio e esolesofhisfeetandalso a circinat tebalanitis. . Rhe eumatoidfactorisnega ativeandhe ehasraised dinflammat torymarker rs. atisthedia agnosis? Wha
A.Behce et'ssyndrom me B.Rheumatoidarthr ritis C.Reiter'ssyndrome e D.Psoria aticarthropa athy E.Ankylosingspondylitis

Answer:c)Reit ter'ssyndro ome. Reit ters syndro ome is urethritis, conju unctivitis, se eronegative e arthritis (c cannot see, , cannot pee e, cannot climb a tree) ). The typic cal patient is i a young man with recent r ureth hritis or dyse entery.The eseronegati ivearthritis sisusuallyamonooroligoarthritis o s. Other features s are anterior uveitis, keratoderm ma blenorrhagica (bro ow n absce esses on palm ms and sole es), mouth ulcers, plan ntar fasciitis s and archil lles tendinit tis (entheso opathy), circinate balanitis (painles ss rash) and d aortic inco ompetence. Managem ment is usua ally with rest tandNSAID Ds.

Kera atodermaBlenorrhagic B cainReiter'ssyndrome e

Dr.Khalid K Yusuf fElzohrySo ohagTeachin ngHospital 2012 116

www.MRCPass.com

Rheumatology

RheumatologyQ112
A 50 year old patient with rheumatoid arthritis has the following full blood count results: Haemoglobin 10.5 g/dL, Platelets 450 x 10^9/L, White Cell Count 8.5 x 10^9/L, MCV103fL. Whichdrugisthelikelycauseofthis?
A.Infliximab B.Ciclosporin C.Leflunomide D.Prednisolone E.Methotrexate

Answer:e)Methotrexate. Methotrexate may lead to macrocytosis as a result of B12 or folate deficiency. It may also be associated with bone marrow suppression, causing leucopenia or thombocytopaenia. Methotrexate may also cause mouth ulcers, stomatitis, cough and dyspnoea.

Dr.KhalidYusufElzohrySohagTeachingHospital2012 117

www.MRCPass.com

Rheumatology

RheumatologyQ113
A32yearoldmanhashadayearshistoryofbilateralhippainsandbackpains.Thereis not past medical history of trauma to the back. Non steroidal antiinflammatory drugs helpedtorelievehissymptoms. Whatisthelikelydiagnosis?
A.Gluteusmediustendonitis B.Osteoarthritis C.Hipfracture D.Sacroilitis E.Osteoarthrosis

Answer:D)sacroilitis. Pain and stiffness in the lower back or buttocks, especially in the morning is typical of sacroilitis. It is typically helped by NSAIDS or steroids. X rays will help to confirm the diagnosis.Itisassociatedwithvariousinflammatorydiseasese.g.ankylosingspondylitis, psoriaticarthritis.

Dr.KhalidYusufElzohrySohagTeachingHospital2012 118

www.MRCPass.com

Rheumatology

RheumatologyQ114
A35year old lady presentswith stiffness,painand swelling of her handsandwrists.On examinationshehasfirmsubcutaneousnodulesoverherelbows,swellingofherwrists, ulnardeviationatthemetacarpophalangealjoints,dinnerforkdeformityandswellingof herproximalinterphalangealjoints. Investigationsrevealanormocytic,normochromicanaemia,elevatedESRandCRP. Which is the most commonly used test which could determine whether she is sero positive?
A.IgAantibody B.IgMantibody C.IgEantibody D.IgDantibody E.IgGantibody

Answer:b)IgMantibody. The patient has features of sero positive rheumatoid arthritis (presence of rheumatoid factor). Rheumatoid factor is a circulating antibody directed against the Fc fragment of immunoglobulin. The antibody may IgM, IgG, or IgA. The commonly employed test detectstheIgMrheumatoidfactor.

Dr.KhalidYusufElzohrySohagTeachingHospital2012 119

www.MRCPass.com

Rheumatology

RheumatologyQ115
A 32 year old man has a 6 month history of dry eyes and mouth. On examination, there was evidenceofkeratoconjunctivitis,parotidgland enlargement and a Schirmer'stestis positive. HisbloodtestsrevealapositiveANA,RoandLaextranuclearantigensarealsopositive. Whatisthediagnosis?
A.SLE B.Rheumatoidarthritis C.Sjogren'ssyndrome D.Wegener'sgranulomatosis E.Pulmonaryeosinophilia

Answer:c)Sjogren'ssyndrome. The patient has Sjogren's syndrome. Sjgren syndrome is a chronic autoimmune disorder characterized by xerostomia (dry mouth), xerophthalmia (dry eyes), and lymphocytic infiltration of the exocrine glands. This triad is also known as the sicca complex. The Schirmer test is probably the only test available in the ED to strongly support or refutesuspicionofSjgrensyndrome.Ateststripoffilterpaperisplacednearthelower conjunctival sac to measure tear formation. A positive test occurs when less than 5 mm offilterpaperiswetafter5minutes. Rheumatoidfactor,ANA,RoandLaantigensarecommonlypresent. As a result of the lymphocytic infiltration, 10% of patients may develop pseudolymphoma, a lymphoproliferative process. Approximately 10% of these patients candevelopnonHodgkinlymphoma(1%ofallpatientswithSjgrensyndrome).

Dr.KhalidYusufElzohrySohagTeachingHospital2012 120

www.MRCPass.com

Rheumatology

RheumatologyQ116
A 38 year old lady presents with myalgia and lethargy. Her blood tests show a positive ANAwithatitreof1:1024andrheumatoidfactorisnegative. The CK is raised at 360 U/l. Extranuclear antigen tests show a negative Ro and negative La,negativeScl70andpositiveribonuclearproteinantibodyat160units. Whatisthediagnosis?
A.Polymyalgiarheumatica B.Polymyositis C.Scleroderma D.Systemiclupuserythematosus E.Mixedconnectivetissuedisease

Answer:e)mixedconnectivetissuedisease. A positive ANA (speckled pattern), raised CK and positive anti RNP antibody suggests mixedconnectivetissuedisease.

Dr.KhalidYusufElzohrySohagTeachingHospital2012 121

www.MRCPass.com

Rheumatology

RheumatologyQ117
A 50 year old man presents to the renal team with uraemic symptoms. He also has markedlyreducedrangeofmovementatthespinewithaSchoberstestof10mm. HisCRPis102mg/dlandESR98mm/hr.Urinedipstickshowsproteinuria++++. He has had treatment with penicillamine in the past for arthritis. He takes regular ibuprofen. Whatisthemostlikelycauseofhisrenalsymptoms?
A.Nephriticsyndrome B.NSAIDnephropathy C.Scleroderma D.ALamyloid E.AAamyloid

Answer:e)AAamyloid. Nephrotic range proteinuria and renal failure in the context of a prolonged untreated inflammatoryresponsesuggestsAAamyloid.SincecirculatingserumAAistheprecursor ofAAamyloiddeposits,reductionoftheprecursorproteinisthemostrationalapproach atpresentforthemanagementofamyloidosis. Prevention of amyloidosis is preferable to treatment of the established disease. Thus aggressivetherapyofrheumaticdiseasessuchasRAandJCAisdesirableinthiscontext.

Dr.KhalidYusufElzohrySohagTeachingHospital2012 122

www.MRCPass.com

Rheumatology

RheumatologyQ118
A 45 year old woman presents with confusion. On examination she was pyrexial, had livedoreticularishadabloodpressureof190/100mmHg. Examinationoftheabdomenrevealedleftflanktenderness. Investigationsrevealed: Hb13.9g/dL whitecellcount6.5x10^9/L plateletcount110x10^9/L serumcreatinine95umol/L urinedipstickanalysis:blood+++,protein+ Whichoneofthefollowingtestsislikelytobepositive?
A.AntiRoantibody B.AntiGBMantibody C.Anticardiolipinantibody D.ASOT E.ANCA

Answer:c)Anticardiolipinantibody. The diagnosis is SLE and antiphopholipid syndrome. The presentation would be consistent with renal vein thrombosis (flank pain and proteinuria). Antiphospholipid syndrome (APS) is a disorder characterized by recurrent venous or arterial thrombosis and/or fetal losses associated with typical laboratory abnormalities. These include persistently elevated levels of antibodies directed against membrane anionic phospholipids(ie,anticardiolipin[aCL]antibody,antiphosphatidylserine). Vascular thrombosis DVT, MI, CVA or miscarriages may occur. Other features are nonthrombotic neurologic symptoms, such as migraine headaches, chorea, seizures, transverse myelitis, GuillainBarr syndrome, thrombocytopenia or hemolytic anemia, Livedoreticularis,AvascularnecrosisofboneandPulmonaryhypertension. Aspirinorwarfarinisrecommendedforpatientswiththromboticsyndromes.

Dr.KhalidYusufElzohrySohagTeachingHospital2012 123

www.MRCPass.com

Rheumatology

RheumatologyQ119
A60yearoldladyhaspolyarthropathyduetoRheumatoidArthtitis. Whichoneofthefollowingmoleculesplaysacentralroleinitspathogenesis?
A.IFNgamma B.Interleukin8 C.TNFalpha D.Endotoxin E.Nitricoxide

Answer:C)TNFalpha. Inthecontextofrheumatoidarthritis,TNFhasinvolvementincytokineregulation,cell recruitment,angiogenesis,andtissuedestruction.HenceantiTNFalphaantibodiessuch asinfliximabareusedintherapy.

Dr.KhalidYusufElzohrySohagTeachingHospital2012 124

www.MRCPass.com

Rheumatology

RheumatologyQ120
A 50 year old woman presents with breathlessness. She has a 2 year history of Raynaud's.Onexamination,shehadtelangiectasiaandtightskinaroundthemouth.Her investigationsshowanESRof70mm/hrandpositiveanticentromereantibodies. Whichofthefollowingisatypicallatecomplicationofthisdisorder?
A.Renalhypertensivecrisis B.Lungmalignancy C.Pulmonaryhypertension D.Myositis E.Mitralregurgitation

Answer:c)Pulmonaryhypertension. The diagnosis is CREST syndrome. Renal hypertensive crisis is more common in diffuse systemic sclerosis and pulmonary hypertension is more common in limited cutaneous scleroderma.

Dr.KhalidYusufElzohrySohagTeachingHospital2012 125

www.MRCPass.com

Rheumatology

RheumatologyQ121
An 18 year old girl developed pulmonary haemorrhage and subsequent acute renal failurerequiringdialysis.Arenalbiopsyshowscrescenticglomerulonephritis. Whichoneofthefollowingantibodiesislikelytobepresent?
A.Antimitochondrial B.Anticentromere C.Antinuclear D.Antiphospholipid E.Antimyeloperoxidase

Answer:e)Antimyeloperoxidase. This patient manifests a pulmonary renal syndrome which is commonly due to an ANCA positive vasculitis. P ANCA which correlates with antimyeloperoxidase (MPO) antibodies, is highly sensitive and specific tow ards rapidly progressive glomerulonephritis and haemorrhagic alveolar capillaritis. Less commonly this could be duetoGoodpasturessyndrome(antiGBMantibodies).

Dr.KhalidYusufElzohrySohagTeachingHospital2012 126

www w.MRCPass.c com

Rheum matology

Rheumatol logyQ122 2
A 65 year old man with chronic leu ukaemia presents with h symptom ms of gout. He was give enAllopurin nol. How wdoesAllop purinolprev ventaccumu ulationofuric u acid?
A.Byinc creasinguric cacidmetabo olism B.Byen nhancingitssolubility s C.Byinh hibitingpurin nesynthesis D.Byinh hibitingpyrim midinesynth hesis E.Byinh hibitingxanthineoxidase e

Answer:E)Byinhibiting i xa anthineoxid dase. Allo opurinol inh hibits xanth hine oxidas se, the enz zyme that catalyzes the conver rsion of hypoxanthine to t xanthine e and of xan nthine to uric u acid. Hy ypoxanthine e and xanth hine are brea akdownpro oductsofpu urine.

Dr.Khalid K Yusuf fElzohrySo ohagTeachin ngHospital 2012 127

www.MRCPass.com

Rheumatology

RheumatologyQ123
A 32 year old woman in the third trimester of her second pregnancy develops acute onsetrightgroinpain. On examination, all right hip movements are painful and she is tender in the right groin and over the greater trochanter. Lumbar spine examination shows an exaggerated lordosis only and there are no neurological lower limb deficits. Initial plain films of the hiparenormal. Themostlikelydiagnosisis:
A.Septicarthritis B.Hipdislocation C.Avascularnecrosisofthehip D.Pregnancyexacerbatingosteoarthritis E.Pseudogout

Answer:c)Avascularnecrosisofthehip. This is a classical presentation of avascular necrosis of the femoral head in pregnancy. Transientosteoporosisofthehipisalsoarecognisedcauseofhippaininpregnancy,but plainradiographswouldusuallyshowmarkedunilateralosteopeniaofthefemoralhead andacetabulum. Thearterialsupplytothefemoralheadiseasilydamagedwithanyfemoralneckfracture displacement. Atraumatic causes of this are : Alcohol abuse, Chemotherapy, Chronic liver disease, Corticosteroids, Gaucher disease, Gout, Hemoglobinopathy (eg, sickle cell disease), Metabolic bone disease, Pregnancy, Radiation, Systemic lupus erythematosus, Vasculitis.

Dr.KhalidYusufElzohrySohagTeachingHospital2012 128

www.MRCPass.com

Rheumatology

RheumatologyQ124
A55yearoldwomanwithpsoriasishassignificantjointpains. Whichoneofthefollowingiseffectiveinthetreatmentofpsoriaticarthropathy?
A.Codeinephosphate B.Methotrexate C.Betainterferon D.Capacitabine E.Buprenorphine

Answer:b)Methotrexate. NSAIDs, sulphasalazine, methotrexate and TNF alpha antagonists are useful in psoriatic arthropathy.

Dr.KhalidYusufElzohrySohagTeachingHospital2012 129

www.MRCPass.com

Rheumatology

RheumatologyQ125
A 65 year old woman attends has a 12 week history of lethargy, neck pains and weaknessinthelowerlimbs. She has a long history of lower back pains and generalised osteoarthritis. She takes diclofenacregularly. On examination there is wasting of her upper limbs. Tone mildly increased in the lower limbs. There is inversion of right supinator reflex, triceps, knee and ankle jerks are brisk bilaterally.Rightplantarisextensorandtheleftisflexor. Investigationsshow: Hb11.6g/l WCC8x10^9/l Plat160x10^9/l ESR73mm CRP12mg/l Na138mmol/l K4.4mmol/l Urea5.8mmol/l Creatinine95umol/l Protein83g/l Albumin32g/l Ca2.33mmol/l Xraycervicalspineshowsextensiveosteophytes Whatisthelikelydiagnosis?
A.Ankylosingspondylitis B.Polymyalgiarheumatica C.Cervicalspondyloarthropathy D.Multiplesclerosis E.Syringomyelia

Answer:c)Cervicalspondyloarthropathy. Cervical spondylosis may present with associated pains in the neck radiating down the armsandback.Theremaybeuppermotorneuronsignsintheupperandlowerlimbs.

Dr.KhalidYusufElzohrySohagTeachingHospital2012 130

www.MRCPass.com

Rheumatology

RheumatologyQ126
A 70 year man presents with right foot drop, hand numbness, fevers, malaise, weight loss,polymyalgiaanddiffusejointpainsfor2months. Onexamination,heappearsunwellandhasatemperatureof38C. Investigationsreveal: Hb8.5g/dL erythrocytesedimentationrate95mm/hr serumcreatinine220mol/L urineanalysis:blood++ urinemicroscopy:whitecells&redcellcastsseen Whichoneofthefollowingisthelikelydiagnosis?
A.Multiplemyeloma B.Antiphospholipidsyndrome C.Takayasu'sarteritis D.Polyarteritisnodosa E.Goodpasture'ssyndrome

Answer:d)polyarteritisnodosa. This patient has a mononeuritis multiplex, fever and nephritic renal involvement suggesting a diagnosis of polyarteritis nodosa. PAN causes transmural necrotizing inflammationofsmallsizedormediumsizedmusculararteries.PANisararecondition. Although the causes are unknown in most cases, there is an association with: Hep B virus, Hep C virus, HIV, Cytomegalovirus, Parvovirus B19 and Human Tlymphotrophic virus. Approximately 20% of patients with classic PAN are positive for PANCA. Steroids (prednisolone) and immunosuppressive (cyclophosphamide) medications form the backbone of therapy. Plasma exchange is useful as a secondline treatment in PAN refractorytoconventionaltherapy.

Dr.KhalidYusufElzohrySohagTeachingHospital2012 131

Vous aimerez peut-être aussi